Back To Search Results

Florida Responsible Controlled Substance and Opioid Prescribing

Editor: Danielle B. Horn Updated: 4/30/2024 11:03:50 PM

Introduction

Pain, both acute and chronic, is one of the most common patient complaints. In an outpatient clinic setting, 1 in 5 patient complaints is related to pain, with over half of all patients consulting their primary care provider for various pain issues. Over one-quarter of the population in the United States experience chronic pain.[1] How a patient's pain is managed can have profound and long-lasting effects on their quality of life. With this emphasis in mind, a trend of increased focus on pain treatment began in the 1990s, leading to an uptick in opioid prescriptions. This trend resulted in widespread abuse, leading to a phenomenon dubbed the opioid epidemic, which prompted both federal and state responses. In the case of Florida, the state legislature passed House Bill 21 (HB 21) in 2018, which significantly restricted opioid prescriptions for non-chronic pain and imposed strict regulations on opioid prescribing for patients with both acute and chronic pain.

Opioid use, misuse, and abuse in the setting of acute and chronic pain pose significant challenges in the United States, particularly in Florida.[2][3] The rate of opioid misuse, addiction, and overdose has been increasing over time in the United States. In 2020, the United States witnessed more than 68,630 deaths due to opioid overdose, constituting 74.8% of all drug overdose deaths and an increase of 31% from the prior year.[158] Of those, 6089 deaths occurred in Florida, indicating a 42% increase (1795 more) from the findings in 2019.[159] Approximately 165,000 individuals in Florida are affected by opioid use disorder, contributing to 2.1 million people nationwide with this condition.[160][4] 

Healthcare professionals must possess a comprehensive understanding of managing patients with both acute and chronic pain. The failure to manage acute and chronic pain appropriately and the possible complication of opioid use disorder secondary to treatment for chronic pain can lead to significant morbidity and mortality. The United States allocates over $100 billion annually on healthcare expenses related to pain management and opioid use disorder.[5] Pain-related expenses exceed the combined costs of cancer, diabetes, and heart disease.[6] 

The International Association for the Study of Pain defines chronic pain as any pain lasting longer than 3 months.[7] Combination therapy for pain includes both pharmacological therapies and nonpharmacological treatment options. Combination therapy yields a more significant reduction in pain compared to single-agent therapy. Escalation of pharmacological therapy should be in a stepwise approach before opioids are considered for treatment. Comorbid psychiatric diseases, especially depression and anxiety, are widespread in patients with chronic pain and should be appropriately addressed and treated.[8] Patients with chronic pain are also at increased risk for suicide.[9] Chronic pain can impact every aspect of a patient's life. Therefore, acquiring the skills to diagnose and effectively manage patients with chronic pain is critical.[10][11]

Unfortunately, studies have revealed a lack of education regarding pain management in most medical schools and training programs.[12] The Association of American Medical Colleges recognized the problem and has encouraged schools to commit to opioid-related education and training by incorporating the CDC and Prevention guidelines for prescribing opioids for chronic pain into the medical school curriculum.[13][14][13]

Appropriate opioid prescribing includes prescribing sufficient opioid medication through regular assessment, treatment planning, and monitoring to provide effective pain relief and enhance the quality of life while avoiding addiction, abuse, overdose, diversion, and misuse. Moreover, clinicians must be aware of both federal and state laws about prescribing controlled substances. To be successful, clinicians must understand appropriate opioid prescribing, assessment, the potential for abuse and addiction, and potential psychological problems, all while remaining in compliance with Florida prescribing requirements. Inappropriate opioid prescribing may involve under-prescribing, overprescribing, or continuing to prescribe opioids when they are no longer effective. These concerns are particularly prominent in patients with chronic pain. To make the challenges even worse, chronic pain patients often develop opioid tolerance and significant psychological, behavioral, and emotional problems, including anxiety and depression associated with the under or overprescribing of opioids.[15]

Clinicians who prescribe opioids face challenges related to medical negligence, including the failure to provide sufficient pain relief, the risk of losing licensure, and the potential for criminal charges if they are suspected of engaging in drug-related diversion or misuse. All clinicians who prescribe opioids require additional education and training to ensure the best patient outcomes and avoid the social and legal entanglements associated with under and overprescribing opioids.[16]

Opioid Knowledge Deficit Among Clinicians

There are substantial knowledge gaps regarding appropriate and inappropriate opioid prescribing, including deficits in understanding current research, legislation, and appropriate prescribing practices. In addition, clinicians often have knowledge deficits that include:

  • Understanding addiction
  • Populations at risk for opioid use disorders
  • Prescription versus nonprescription opioid use disorders
  • The misconception that use disorders and tolerance to opioids are synonymous
  • The belief is that opioid use disorders are solely a psychological problem, whereas, in reality, it is likely a complex psychological and physiological issue related to chronic, painful disease symptomatology.

Prescribing opioids has resulted in significant societal challenges that can only be resolved through comprehensive education and training for all healthcare team members.[13]

Definitions

  • Use disorder: Use of a legal or illegal substance or maladaptive pattern of substance use for a nonmedical purpose other than pain relief, typically to alter a patient's state of consciousness.[17]
  • Addiction*: Pursuit of a chemical substance to seek relief or reward associated with diminished control over drug use, compulsive use, craving, or continued use despite adverse consequences.
  • Dependence: Physiological reliance on a drug resulting in a withdrawal syndrome with cessation or reduction in the amount of drug administered.[18]
  • Diversion: Transferring a controlled substance from an authorized person's use to an unauthorized person's use for distribution or possession.
  • Misuse: Use of a medication in a manner that deviates from the prescribed guidelines.
  • Pseudoaddiction: Pursuit of additional medication due to poor pain control, with the cessation of drug-seeking behavior upon achievement of appropriate pain relief.
  • Tolerance: The lessened effect of a substance after being exposed to that substance over a period of time, with the need to escalate doses to achieve the same effect.[19]

Clinicians' understanding of addiction is sometimes confusing, inaccurate, or inconsistent due to the broad range of perspectives encountered when dealing with patients experiencing addiction. Although a knowledge gap is present among clinicians, it is equally prevalent in both politicians writing laws and law enforcement officials attempting to enforce the laws. In addition, payers are responsible for the expenses of evaluating and treating addiction and its associated use disorders. Persistent lack of education and the use of obsolete terminology continue to contribute to a societal lack of understanding for effectively dealing with addiction challenges.

In the past, the American Psychiatric Association's Diagnostic and Statistical Manual of Mental Disorders defined addiction, substance abuse, and substance dependence separately. Over time, the manual has eliminated these terms and now uses substance use disorder, ranging from mild to severe.[13]

*The American Society of Addiction Medicine describes addiction as a treatable chronic disease that involves environmental pressures, genetics, an individual's life experiences, and interactions among brain circuitry. Individuals who develop addiction to opioids or other medications often exhibit compulsive behaviors that result in dangerous consequences. Although the following characteristics should not be used as diagnostic criteria due to variations among addicted individuals, the American Society of Addiction Medicines identifies 5 key addiction behaviors:

  • Craving for drug or positive reward
  • Dysfunctional emotional response
  • Failure to recognize significant problems affecting behavior and relationships
  • Inability to consistently abstain
  • Impairment in control of behavior

Etiology

Register For Free And Read The Full Article
Get the answers you need instantly with the StatPearls Clinical Decision Support tool. StatPearls spent the last decade developing the largest and most updated Point-of Care resource ever developed. Earn CME/CE by searching and reading articles.
  • Dropdown arrow Search engine and full access to all medical articles
  • Dropdown arrow 10 free questions in your specialty
  • Dropdown arrow Free CME/CE Activities
  • Dropdown arrow Free daily question in your email
  • Dropdown arrow Save favorite articles to your dashboard
  • Dropdown arrow Emails offering discounts

Learn more about a Subscription to StatPearls Point-of-Care

Etiology

Pain Divisions

Pain can be divided into various categories and types, including neuropathic, nociceptive, musculoskeletal, inflammatory, psychogenic, and mechanical.[10][20] In addition, pain is often classified based on chronicity. Generally, pain lasting less than 3 months is defined as acute pain, and pain lasting more than 3 months is termed chronic pain.[7] This distinction is crucial for prescribers in Florida. HB 21 bases its prescribing regulations on this distinction between acute and chronic pain. Acute pain, as defined by the bill, is the normal, predicted, physiological, and time-limited response to an adverse chemical, thermal, or mechanical stimulus associated with surgery, trauma, or acute illness. Notably, this definition excludes pain related to:

  • Cancer
  • Terminal conditions
  • Palliative care to provide relief for symptoms related to an incurable, progressive illness or injury
  • A severe traumatic injury with an Injury Severity Score of 9 or greater

All other pain not fitting this time-limited definition is classified as chronic pain.[161]

Causes of Pain

Both acute and chronic pain can be multifactorial and not necessarily fit neatly into a single pain type, such as neuropathic or inflammatory. However, this complexity is especially true in the development of chronic pain. Most patients with chronic pain report experiencing more than one type of pain.[21] For example, a patient with chronic back pain may also have fibromyalgia. A significant percentage of patients experience major depressive disorder and generalized anxiety disorder. Over 67% of patients with chronic pain experience a comorbid psychiatric disorder.[22]

Causes of Opioid Use Disorder and Associated Morbidity and Mortality

Although opioids effectively manage various pain types, their use is linked to morbidity, mortality, and abuse. Opioid use disorder stems from medical, psychological, social, and economic factors.

Medical and Social Factors

Twenty years ago, a national shift occurred when clinicians were encouraged to treat pain more aggressively. The pain was considered the fifth vital sign, and clinicians who undertreated non-cancer pain potentially incurred liability, patient complaints, and licensing board complaints. At the same time, oxycodone was introduced and touted to have low addiction potential. Furthermore, patients and clinicians concluded that prescription opioids were better and safer compared to street drugs. The result was more than a decade of growth in prescription opioid use with severe adverse consequences for individual patients and communities.[23][24][24]

Economic Factors

Opioid use has increased due to direct marketing to consumers, promotion to physicians and pharmacists by sales representatives, and the illicit production and distribution of opioids. Due to the possibility of making enormous profits, some unethical healthcare professionals developed pill mills where opioid prescriptions were given with little or no documentation. This scenario was widespread in Florida. By the late 2000s, the DEA considered Florida to be the epicenter of pill mills in the United States and was the target of multiple extensive DEA operations.[162]

According to DEA estimates from 2009, Florida was the leading distributor of oxycodone nationwide, leading second-ranked California by 40%. In 2010, the DEA stated that 90 of the top 100 oxycodone distributors originated in Florida.[25][163] As a result, the DEA closed nearly 600 pain clinics in Florida between 2010 and 2012.[163][164] As these prescription sources have gradually diminished, individuals with opioid use disorders have turned to alternative sources, and illicit fentanyl and heroin use has made a resurgence.[26][27][28][29]

Manufacturers

Since the early 2000s, many lawsuits have been filed against opioid manufacturers and distributors, with claims that manufacturers did not provide adequate warnings regarding the addiction potential of opioids or, worse, knowingly withheld information about the risks of opioids.[30]

Prescribers

Prescribers who exceed practice guidelines by prescribing higher than normal doses, long-acting formulations, and large volume prescriptions with limited follow-up for chronic, non-cancer pain may increase the risk of opioid misuse or abuse and associated morbidity and mortality among patients. In addition, they may face severe sanctions from their affiliated boards.[31]

Dispensers

Interprofessional collaboration, often mandated by states, is crucial for promoting safe patient care. Pharmacists and prescribers should have an open dialogue regarding prescriptions, and pharmacists must be able to obtain further information from prescribers regarding prescriptions that seem unreasonable. Pharmacists have access to state opioid use databases and may contact prescribers if they have concerns about controlled substance prescriptions for patients.[32][33]

Epidemiology

Frequency of Pain

The incidence of acute pain is challenging to determine, as it often manifests as a symptom rather than a diagnosis. However, it is a common general complaint and the leading chief complaint in the emergency department.[34] More than three-fourths of patients who present to the emergency department cite pain as their primary concern.[35] According to the Centers for Disease Control (CDC), about 20.4% of people older than 18 in the United States experienced chronic pain in 2016.[36] Furthermore, approximately 8% of adults in the United States experienced chronic pain severe enough to result in repeated limitations on integral life and work activities.

Chronic pain was found to be more prevalent in women, older adults, adults with a lower socioeconomic status, those with public health insurance, and adults without education beyond middle school. Among chronic pain sufferers, chronic regional pain affects 11.1%, chronic back pain accounts for 10.1%, leg and foot pain for 7.1%, arm and hand pain for 4.1%, and headache for 3.5%. Reports indicate widespread pain in 3.6% of chronic pain patients.[21]

Chronic pain costs approximately $560 billion annually in lost productivity and medical treatment.[36] Research has shown that the lifetime prevalence of chronic pain patients attempting suicide is between 5% and 14%; suicidal ideation was approximately 20%.[37] Of the chronic pain patients who committed suicide, 53.6% died from firearm-related injuries, whereas 16.2% died from an opioid overdose.[38]

The incidence of chronic pain is increasing due to the prevalence of obesity-related pain conditions, increased survival of trauma and surgical patients, an aging population, and heightened public awareness of pain as a treatable condition. Of note, older patients have been shown to receive up to 25% fewer pain medications compared to the average population.[39]

Frequency of Opioid Use, Morbidity, and Mortality

In the past, healthcare professionals in the United States rarely prescribed opioids for any condition except chronic cancer pain. This approach began to change in the 1990s. Dr. James Campbell addressed the American Pain Society in 1995 and urged healthcare professionals to treat pain as the fifth vital sign.[40] Simultaneously, drug companies aggressively marketed and promoted these medications while grossly downplaying the risks, particularly addiction.[41] By 2004, the extended-release form of oxycodone had become the most abused drug in the United States.[42] The United States currently consumes more than 80% of all opioids produced worldwide.[43][44] With increased use, concurrent problems have developed, and the number of individuals abusing opioid analgesics has increased dramatically.[45][46]

Pathophysiology

The pathophysiology of acute and chronic pain is complex and not completely understood. However, it is known that many factors contribute to the interpretation of pain, especially the conversion of chronic pain, including inflammation, modulation of signals in the central nervous system by chemicals such as endorphins and monoamines, and psychological factors. Many theories attempt to explain the complex interplay that creates the experience of pain, including the gate control theory, the neuromatrix model, and the biopsychosocial model, among others. In general, the following provides an overview of the current understanding of pain and its pathophysiology.[10][20][47]

Acute Pain

Acute pain arises from activating peripheral pain receptors and specific A-delta (A-δ) and C-sensory nerve fibers (nociceptors). Acute pain commonly occurs in response to tissue injury and is typically classified based on duration, typically pain lasting for 3 months or less. As mentioned above, acute pain, as defined by Florida HB 21, is the normal, predicted, physiological, and time-limited response to an adverse chemical, thermal, or mechanical stimulus associated with surgery, trauma, or acute illness and specifically excludes pain related to cancer, terminal illnesses, palliative care for incurable conditions or illnesses, and severe traumatic injuries. 

Chronic Pain

Chronic pain is characterized by its duration, typically lasting longer than 3 months.[48] Some define chronic pain as pain that persists beyond normal healing time. However, there are cases in which normal healing has not occurred, and the pain is still classified as chronic, such as pain associated with inflammatory disease or metastatic carcinoma. Chronic pain may result from the persistent activation of pain fibers secondary to ongoing tissue injury. In addition, chronic pain may arise from ongoing damage to or dysfunction of the peripheral or central nervous system, also known as neuropathic pain.

Nociceptive Pain

Nociceptive pain receptors can be classified as either somatic or visceral.

Somatic pain receptors are located in the subcutaneous tissues, fascia, connective tissues, endosteum, periosteum, joint capsules, and skin; stimulation results in typically well-localized pain.

Visceral pain receptors are located in the viscera and connective tissue. Visceral pain may be due to infection, stretch, distension due to fluid or gas, or compression of hollow organs by tumors. This pain is deep, cramping, poorly localized, and may be referred to a remote site. Visceral pain from connective tissue injury or organ capsules may be more localized and sharp.

Neuropathic Pain

Neuropathic pain typically arises due to damage to peripheral or central nervous system structures. Some common examples include diabetic neuropathy, postherpetic neuralgia, or lumbar radiculopathy (that is, sciatica). This pain is typically described as numbness, tingling, shooting, or burning.

Psychological Factors

Psychological factors play a highly variable role in modulating the intensity of pain. Culture, emotions, and thoughts affect an individual's perception of pain. Patients with chronic pain may also have psychological stress. Chronic pain patients often have comorbid mental health disorders and have a higher incidence of anxiety and depression compared to the general population.[49][50] As a result of some syndromes being characterized as psychiatric disorders, such as somatic symptom disorders defined by self-reported pain, patients are often mischaracterized as having a psychiatric disorder and do not receive appropriate care. Pain may interfere with cognitive attention, concentration, thought content, and memory.

Multifactorial Pain Syndromes

Many pain syndromes are multifactorial. For example, cancer and chronic low back pain syndromes have a nociceptive component and may also involve neuropathic pain due to nerve damage.

Pain Modulation and Transmission

Pain fibers enter the spinal cord at the dorsal root ganglia and synapse in the dorsal horn. Fibers then cross to the contralateral side and travel up the lateral columns, reaching the thalamus and, finally, the cerebral cortex. Repetitive stimulation from prolonged pain can sensitize neurons in the posterior horn of the spinal cord so that a minimal peripheral stimulus results in pain.

Peripheral nerves and those at other levels of the central nervous system are sensitized, resulting in long-term synaptic changes in cortical receptive fields. This alteration can result in exaggerated pain perception. This process of chronic afferent input causes increased sensitivity and lower pain thresholds, with the remodeling of the central nociceptive pathways and receptors termed central sensitization. The consequences may include:

  • Allodynia: Nonpainful stimulus perceived as painful
  • Hyperalgesia: Excessive pain response to a normally painful stimulus

Mechanism of Opioids

Any compound that binds to an opioid receptor is considered an opioid. This category includes endogenous opioids such as endorphins, enkephalins, and dynorphins, which are released in response to noxious stimuli, and prescription and illicit opioids, all of which bind opioid receptors. Opioid analgesics act on the 3 major opioid receptors—mu (μ), delta (δ), and kappa (κ)—and several minor classes of opioid receptors at both pre- and postsynaptic terminals. Most commonly, opioid analgesics are potent μ-receptor agonists.[51]

The μ-receptors are believed to mediate analgesia, respiratory suppression, bradycardia, physical dependence, gastrointestinal dysmotility, and euphoria. Kappa agonism can lead to hallucinations, miosis, and dysphoria. The delta receptor likely has pain control and mood modulation effects, but some have suggested that μ-agonism is necessary for the δ-receptor to exert strong analgesic effects.[52][53]

All opioid receptors are G-protein–coupled receptors that send signals through a second messenger, cyclic adenosine monophosphate (cAMP), to activate protein kinases. When activated, cAMP kinase can lead to increased excitability, catecholamine synthesis, and alterations in gene transcription proteins and gene expression. Opioid receptors are distributed widely in the brain, spinal cord, peripheral neurons, and digestive tract. When opioids bind to their receptors on presynaptic terminals of pain fibers, such as C-fibers and A-δ fibers, voltage-gated calcium channels are inhibited, decreasing calcium ion entry into the cell and reducing cAMP levels. As a result, there is a decrease in nociceptive fibers' presynaptic release of pro-nociceptive neurotransmitters such as glutamate, calcitonin gene–related peptide, and substance P, resulting in diminished nociception.[54] Opioids also enhance potassium efflux from the cell, resulting in the hyperpolarization of cell membranes and inhibition of postsynaptic neuronal activity.

Opioids decrease the release of gamma-aminobutyric acid (GABA), resulting in an increase in the firing of dopaminergic neurons. An increase in dopamine levels in the nucleus accumbens plays a role in pleasure, reward, addiction, and analgesia.[55] Certain opioids may also act as N-methyl-D-aspartate (NMDA) receptor antagonists, such as methadone, resulting in the activation of serotonin and norepinephrine descending analgesic pathways.[54]

Opioids may be pure μ-receptor agonists, such as morphine, fentanyl, and oxycodone; agonists or antagonists, also known as mixed agonists or antagonists; or partial agonists, such as buprenorphine. 

Toxicokinetics

Opioids exhibit a wide range of durations and intensities of effect. For example, remifentanil has a half-life of around 10 to 20 minutes, whereas methadone has a half-life between 8 and 60 hours (with an average half-life of about 24 hours).[56] The half-life of methadone is complicated and variable based on the duration of use and the number of doses given. The uptake and effect of opioids also vary depending on the route of administration used, with examples including fentanyl patches or long-acting oral formulations of oxycodone and morphine.

Some opioids, such as diphenoxylate and loperamide, have almost no effect other than the suppression of bowel motility. Opioids such as methadone can significantly prolong the QT interval. In addition, opioids can sometimes precipitate serotonin syndrome, especially when administered to patients already taking various psychoactive medications, such as antidepressant medications, including selective serotonin reuptake inhibitors. This risk is particularly true for tramadol and meperidine. There is an evolving body of knowledge suggesting that the intensity and quality of response to opioids can vary significantly among patients, which can be unrelated to tolerance. This association is likely linked to genetics but is not currently well-characterized.[57][46][58]

History and Physical

History

History should include the onset of pain, description, mechanism of injury if applicable, location, radiation of pain, quality, severity, factors contributing to relief or worsening of the pain, frequency of the pain, duration of the pain, and any breakthrough pain. A verbal numeric rating scale (VNRS) or number scale (0 to 10) can aid in assessing pain intensity. Furthermore, associated symptoms such as muscle spasms or aches, temperature changes, restrictions to range of motion, morning stiffness, weakness, muscle strength, sensation changes, and hair, skin, or nail changes should be assessed.

In addition to the patient's symptoms, the impact of the pain on day-to-day function should be discussed, and daily living activities should be reviewed. Understanding how pain, particularly chronic pain, affects the patient's quality of life, relationships, hobbies, work, activities of daily living, sleep, mood, and exercise is essential.

A multidimensional assessment of a patient's pain and its severity should be completed and can include assessment guides such as a Pain, Enjoyment, General (PEG) activity tool; the 4-item Patient Health Questionnaire (PHQ-4 ); or the Defense and Veterans Pain Rating Scale (DVRPS).[59][60][61][62] For children, behavioral observation scales are often used to assess pain.[63] Age-based pain rating scales can be used. Visual analogs are also often implemented, with pictures depicting varying degrees of distress. The Pediatric Pain Questionnaire and the Adolescent and Pediatric Pain Tool are commonly used to assess pediatric pain.[64] As children reach adolescence, they typically can rate their pain on a numerical scale, similar to adults.[65] 

Assessing pain in nonverbal children with neurologic impairment is particularly challenging. Caregivers are often required to help determine changes in the patient's behavior. Symptoms such as grimacing, moaning, increased muscle tone, crying, arching, and atypical behavior such as aggressive behavior are crucial indicators to monitor in this population. Observational pain assessment tools are used in populations who cannot self-report. The observational findings used in such an assessment tool include facial expression, fussiness, distractibility, ability to be consoled, verbal responsiveness, and motor control. For infants and young children, the Revised Face, Legs, Activity, Cry, Consolability (r-FLACC) tool is commonly utilized.[63][66][67][68] Although the NAPI tool is highly rated, several other validated tools are available and commonly used.[69][70][71][66]

The Brief Pain Inventory (BPI) can be used to assess patients' beliefs about pain and its impact on their daily lives.[72][73] Separately, the McGill Pain Questionnaire (SF-MPQ-2) includes a body map to identify the location of pain in the human body, a questionnaire about previous use of pain medication, and past experiences with pain.[74] Neuropathic pain can be assessed using the Neuropathic Pain Scale to follow responses to therapy.

When prescribing controlled substances for pain management, clinicians should obtain the patient's detailed history of substance abuse, psychiatric disorders, or other medical or social factors that could lead to an increased likelihood of abuse or misuse of medication. Before prescribing controlled substances for any pain, whether acute or chronic, it is crucial to conduct a comprehensive evaluation, including obtaining a detailed history, performing a thorough physical examination, reviewing the patient's medical records, and querying the state or regional Prescription Drug Monitoring Program (PDMP). These steps are required when prescribing controlled substances in Florida.

Physical

A thorough physical examination, including musculoskeletal, neurologic, and psychiatric examinations, should be conducted, along with a targeted examination of the specific area experiencing pain.

Evaluation

Pain Assessment

Standard blood work and imaging are not necessarily indicated for chronic pain that has been previously assessed. However, clinicians may order them when specific causes of pain are suspected and when there is a worsening of chronic pain or a change in symptoms. Ordering blood work at regular intervals when patients chronically use pain medications, such as nonsteroidal anti-inflammatory drugs (NSAIDs), acetaminophen, and other drugs, is advised. When managing patients with opioid therapy for chronic pain, it is advisable to obtain random urine toxicology screening at the start of implementation of opioid treatment and then at least annually to monitor compliance and exclude the use of nonprescription drugs, aligning with current recommendations from the CDC.

Psychiatric disorders have the potential to intensify pain signals, thereby worsening the symptoms of pain.[75] Chronic pain is associated with higher rates of mental health disorders compared to the general population.[76] Some studies have shown that up to 75% of patients experience an overlap of chronic pain and mental illness. Most notably, chronic pain patients have a higher prevalence of depression, anxiety, and substance use disorders.[77] Symptoms, such as weight or appetite changes, fatigue, or sleep changes, commonly associated with major depressive disorder overlap with those of chronic pain. These symptoms can make their pain worse over time. Patients with chronic pain are at an increased risk of suicide and suicidal ideation.[37][38]

Furthermore, comorbid psychiatric disorders, such as major depressive disorder, can significantly delay the diagnosis of pain disorders.[78] There are twice as many prescriptions for opioids prescribed each year to patients with underlying pain and a comorbid psychiatric disorder compared to patients without such comorbidity.[79] Simultaneous screening for depression is recommended for patients with chronic pain. The Minnesota Multiphasic Personality Inventory-II (MMPI-2) or Beck's Depression Scale are the 2 most commonly used tools for this purpose, with the MMPI-2 being more frequently used for patients with chronic pain.[80][81]

Prescribing Opioids

In Florida, HB 21 sets strict regulations on prescribing opioids. All prescribing practitioners eligible to prescribe controlled substances must complete a 2-hour training course covering safe and effective prescribing practices, which must be updated every 2 years. Consulting the state's PDMP, known as Electronic-Florida Online Reporting of Controlled Substance Evaluation (EFORCSE), is also mandatory before prescribing.[165]

Before prescribing opioids, a complete and detailed patient history should include the following:

  • Indication requested for pain relief
  • Location, nature, and intensity of pain
  • Prior pain treatments and response
  • Comorbid conditions
  • Potential physical and psychological pain impact on function
  • Family support, employment, and housing
  • Leisure activities, mood, sleep, substance use, and work
  • Emotional, physical, or sexual abuse

When considering opioids, the risks of abuse, addiction, adverse drug reactions, overdose, and physical dependence should be weighed. A psychiatrist or addiction specialist should be consulted if there are any particular concerns, such as a history of substance abuse. If the patient is currently abusing, it may be prudent to delay prescribing until the patient is involved in an addiction treatment and monitoring program.

The World Health Organization (WHO) Analgesic Ladder should be followed rather than starting an opioid without trialing alternative therapies either before or alongside opioid medications.[82] Patients should be offered multimodal analgesic regimens starting with nonopioid analgesics such as acetaminophen, NSAIDs, or adjuncts, particularly anticonvulsants or analgesic antidepressants, when appropriate. Nonpharmacologic treatments, including but not limited to physical therapy, psychological therapy, and soft-tissue modalities, may also be considered. 

Addiction Risk Assessment

Clinicians should gather information from various sources, including patient history, physical examination, input from family members, data from the state prescription monitoring program, and screening tools, to assess the risk of developing an untoward behavioral response to opioids. Patients can be categorized into 3 risk levels based on their responses to screening tools.

  • Low risk: standard monitoring, vigilance, and care.[45]
  • Moderate risk: additional level of monitoring and more frequent healthcare professional contact.[83]
  • High risk: intensive and structured monitoring, frequent follow-up contact, consultation with an addiction psychiatrist, and limited monthly prescription of short-acting opioids.[84]

Personal or family history of substance abuse, current substance abuse, poor social support, family or social environments that lend themselves to misuse, younger age, and comorbid untreated psychiatric diseases are some risk factors for opioid misuse and addiction.[85]

Assessment Tools

Screening tools help determine the risk level, degree of monitoring, and treatment plan structure. However, their validity is not yet supported in the literature. Some examples of questionnaires that can be helpful in the evaluation of the potential for opioid abuse include:[84][86]

  • Brief Intervention Tool
  • Cut-down, Annoyed, Guilty, and Eye-opener (CAGE), CAGE-AID, and CAGE-Opioid
  • Current Opioid Misuse Measure (COMM)
  • Diagnosis, Intractability, Risk, and Efficacy (DIRE) Tool
  • Mental Health Screening Tool
  • Opioid Risk Tool (ORT)
  • Pain Assessment and Documentation Tool (PADT)
  • Screener and Opioid Assessment for Patients with Pain-Revised (SOAPP-R)

Urine Drug Tests

Urine drug tests assess the use of the medication prescribed and detect unsanctioned drug use. The CDC recommends drug testing before starting opioid therapy and at least annually.

Based on risk assessment, more frequent testing should be conducted, with a focus on identifying specific drugs.[87] If urine test results suggest aberrant opioid use, lack of use, or use of other nonprescribed controlled or illicit substances, it is essential to address the issue with a positive, supportive approach and document the discussion.

Prescription Drug Monitoring Programs

PDMPs are now established to varying degrees in all states and most territories. In Florida, the PDMP is commonly known as E-FORSCE. PDMPs enable collaboration between prescribers and dispensers to decrease drug abuse and prevent unauthorized access to prescription opioids. The PDMP regulations are state-based and, therefore, vary in application.

The key benefits of PDMPs include:

  • Assists in monitoring outpatient prescriptions.
  • National decrease in the number of prescriptions provided by multiple healthcare professionals (avoids doctor shopping).
  • Encourages collaboration between prescribers and pharmacists as an interprofessional team.
  • Assists regulatory boards, Medicaid, medical examiners, law enforcement, and research organizations in gathering data on the effectiveness and enforcement of provisions.
  • Some states, including Florida, are participating in PMP Interconnect, allowing prescription information to be shared across state lines.

In Florida, it is mandatory for all prescribers to consult E-FORSCE every time they issue a controlled prescription, regardless of the quantity and whether the script is for acute or chronic pain. The current CDC recommends that prescribers review the PDMP when initiating opioid therapy, every three months thereafter, and each time they write a prescription.[88]

Checklists For Prescribers and Dispensers

Prescribing or Dispensing

  • The prescribed dose, frequency, and duration are consistent with the indication; long-acting opioids for acute pain, using multiple short-acting opioids concurrently, or multiple long-acting opioids concurrently should be avoided.
  • Age, weight, height, and sex considered.
  • Evaluate for potential drug interactions.
  • Evaluate for potential allergic reactions.
  • The patient is informed and understands the risks and benefits, including addiction, abrupt halting, use of power equipment, adverse effects, respiratory depression, avoiding sharing, storage, and theft protection.
  • Medication use agreement in place.
  • Instructions for storage and disposal of unused opioids.

VIGIL

  • Verification: Is this a responsible opioid user?
  • Identification: Is the identity of this patient verifiable?
  • Generalization: Do we agree on mutual responsibilities and expectations?
  • Interpretation: Do I feel comfortable giving this person access to controlled substances?
  • Legalization: Am I acting legally and responsibly?

Patient Validation and Red Flags

  • Check government-issued photo ID
  • Only drugs prescribed are controlled substances
  • Early refills
  • Insurance present but pays cash
  • Lost prescriptions
  • Remote address
  • Multiple prescribers
  • Unrealistic expectations
  • PDMP or urine drug screen yields concerning results

Prescriber Validation and Red Flags

  • The prescriber writes the same drug, dose, frequency, and length despite differences in age, height, sex, and weight
  • The prescriber is outside the patient's geographic area
  • The prescriber writes prescriptions that are outside the scope of their practice
  • The prescriber writes for a combination of opioids, benzodiazepines, or muscle relaxants
  • Excessive drug quantities are provided
  • There are multiple prescriptions from the same healthcare professional
  • The healthcare professional does not take insurance

Prescription Validation and Red Flags

  • Eraser marks
  • Handwriting irregularities
  • Late-night and weekend prescription drop-offs
  • Prescription irregularities
  • Federal and state-controlled substance number irregularities
  • Evidence of being turned away by another pharmacy
  • Misspellings

How to Handle Patient Interactions When Declining to Prescribe or Dispense Controlled Substances 

  • Provide news to the patient in a calm manner
  • Do not accuse of wrongdoing
  • Report concerning prescription(s) to the authorities

Treatment / Management

Healthcare professionals who treat patients with acute or chronic pain should understand best practices in opioid prescribing; pain assessment methodologies; pain management modalities, including multimodal approaches to treating pain; appropriate use of opioids for pain control, and local and state laws about prescribing opioids. Pharmacological and nonpharmacological approaches should be assessed. For moderate-to-severe acute pain, patients can be prescribed a short course of opioids. Under Florida law, a prescription for acute pain can be for no more than 3 days for an opioid listed in Schedule II. This limit can be increased to 7 days if determined to be medically necessary with supporting documentation.[166]

However, patients with a chronic pain diagnosis are not bound by the same limit. Patients with moderate-to-severe chronic pain assessed and treated with nonopioid therapy without adequate pain relief may be candidates for opioid therapy if the benefits outweigh the risks. Initial treatment should be a trial of the lowest effective dose with plans for discontinuation if it is decided that the risks outweigh the benefits. Initial implementation of opioid therapy should not necessarily be considered a definitive course of long-term treatment. The CDC has issued updated guidance on prescribing opioids for chronic pain. These guidelines address various facets of opioid prescription, including when to initiate or continue opioids for chronic pain, opioid selection, dosage, duration, follow-up, and discontinuation. In addition, the guidelines emphasize the importance of assessing the risk of opioid misuse and addressing potential harms associated with opioid use.[88] 

Pain Management Referral

Patients with chronic pain who do not respond to initial therapy may be referred to a pain management clinic. There are multiple pharmacological, adjunct, nonpharmacological, and interventional treatment options for managing chronic, severe, and persistent pain.

Nonpharmacological options: The list of nonpharmacological therapies for pain is extensive. Nonpharmacological options include heat and cold therapy, massage therapy, cognitive behavioral therapy, relaxation therapy, biofeedback, group counseling, ultrasound stimulation, acupuncture, aerobic exercise, chiropractic therapy, physical therapy, occupational therapy, and transcutaneous electrical nerve stimulation. These options exhibit varying levels of evidence regarding their efficacy. For example, cognitive behavioral therapy has some beneficial effects on pain, disability, and distress reduction in patients with chronic pain based on a Cochrane review. However, the data supporting the efficacy of transcutaneous electrical nerve stimulation for the treatment of chronic pain have been inconclusive.[89][90](A1)

Interventional techniques can also be utilized to treat acute and chronic pain. Nerve blocks are routinely performed both for pain prevention, that is, preoperatively, or for treating acute pain, that is, postoperatively, or chronic pain. Some commonly performed procedures for treating chronic pain include trigger point injections, nerve blocks, radiofrequency or cryo-ablations, intra-articular injections, and epidural steroid injections. Neuromodulation procedures, including spinal cord stimulation, peripheral nerve stimulation, and intrathecal drug delivery systems, are also options, particularly in patients who have exhausted more conservative measures. The Food and Drug Administration has approved spinal cord stimulation to manage chronic intractable pain of the trunk and limbs associated with failed back surgery syndrome, complex regional pain syndrome, and neuropathy.[91]

Pharmacological options and risks/benefits: The range of pharmacological options available for treating pain is extensive—NSAIDs, acetaminophen, local anesthetics, corticosteroids, anticonvulsants, antidepressants, topical analgesics, skeletal muscle relaxants, NMDA receptor antagonists, alpha-2 adrenergic agonists, and opioids.

Musculoskeletal pain conditions involve both central and peripheral sensitization mechanisms.[92] The treatment of such pain should be based on a stepwise approach, incorporating a combination of nonopioid analgesics, opioids, and nonpharmacological therapies. First-line therapy should be acetaminophen or NSAIDs, both of which can be effective for osteoarthritis and back pain.[93][94] However, NSAIDs are relatively contraindicated in patients with peptic ulcer disease, coronary artery disease, renal disease, concurrent anticoagulation use, and preexisting platelet defects.[95] There is limited evidence to determine which NSAID to use over another. However, it is crucial to consider the intrinsic pharmacokinetic, pharmacodynamic, tolerability, and formulation-specific factors of various NSAID classes when making a prescription choice for the patient.[96](A1)

If neuropathic pain is a component of the patient's chronic pain syndrome, a combination of multiple pharmacological therapies may be necessary. Less than half of patients with neuropathic pain achieve adequate pain relief with a single agent.[97] The initial treatment of neuropathic pain is often with gabapentinoids, such as gabapentin and pregabalin. These drugs inhibit voltage-gated calcium channels by binding to the alpha-2 δ subunit and are recommended for neuropathic pain syndromes, including postherpetic neuralgia, diabetic neuropathy, and mixed neuropathy.[98] Other anticonvulsants, such as carbamazepine and oxcarbazepine, are beneficial for particular pain syndromes, such as trigeminal neuralgia.[99](A1)

Antidepressants in the class of serotonin-norepinephrine reuptake inhibitors and tricyclic antidepressants are other options for pain management. Antidepressants may be beneficial in treating neuropathic pain, central pain syndromes, and chronic musculoskeletal pain. Duloxetine is a serotonin-norepinephrine reuptake inhibitor that can be used to treat chronic pain from osteoarthritis, fibromyalgia, low back pain, and diabetic peripheral neuropathy.[100] Venlafaxine and milnacipran are other serotonin-norepinephrine reuptake inhibitors that may be effective treatment options for specific pain conditions.[101] Commonly used tricyclic antidepressants include nortriptyline, amitriptyline, and desipramine. These medications may require 6 to 8 weeks to achieve their desired effect.[102](A1)

Adjunctive topical agents can be used to manage chronic pain and allow for fewer systemic adverse effects compared to parenteral or oral administration. Common examples include lidocaine; topical NSAIDs, such as diclofenac; capsaicin; and methyl salicylate-menthol. Compounded topical medications, which typically include a mixture of 3 or more single drugs, provide a more customized therapy regimen. However, the studies are limited due to the preparation of these medications and the specific pain conditions for which they are used.[103]

In addition, botulinum toxin can also be an option for chronic pain and disorders of muscle spasms, including myofascial pain syndrome, headaches, temporomandibular joint disorders, and neuropathic pain.[104]

Opioids serve as a primary treatment for moderate-to-severe acute pain but are considered a second-line option for chronic pain syndromes. However, they may be warranted for pain management in patients with severe persistent pain refractory to other treatments or especially pain due to malignancy. In cases where nonopioid therapies have failed or immediate pain relief is needed, opioid therapy can be considered for managing neuropathic pain or while titrating first-line medications.[102] Adverse effects of opioids are significant and may include opioid-induced hyperalgesia, constipation, dependence, sedation, immunosuppression, osteoporosis, and neuroendocrine dysfunction.[105] For patients on medium- and long-term use of opioids, there is an estimated 78% risk of an adverse reaction to opioids, such as constipation or nausea, whereas there is a 7.5% risk of developing a severe adverse reaction ranging from immunosuppression to respiratory depression.[106] (B3)

Patients must make an informed choice before starting opioid treatment after discussing the risks, benefits, and alternatives to opioids.[107] The risk of overdose increases directly with increasing doses of opioids. Compared to patients on less than 20 morphine milligram equivalents (MME) per day, those on 50 to 99 MME/d double their risk of overdose; this factor increases up to 9 with ≥100 MME/d.[108] 

When initiating opioids, clinicians should start with a dose as low as reasonably achievable (ALARA) to provide analgesia without significant adverse effects.[109] Moreover, the correct opioid selection is paramount. Each opioid medication has specific traits which may make it better suited for certain situations. As such, clinicians should have a basic understanding of the pharmacological properties of commonly used opioids.

Chemical Classes of Opioids

Opioids can be classified into 4 chemical classes.

  • Alkaloids extracted from poppy seeds or phenanthrene opioid agonists
    • Morphine, codeine
    • Semi-synthetic derivatives: Oxycodone, oxymorphone, hydromorphone, and hydrocodone
  • Synthetic phenylpiperidines: Meperidine, fentanyl, sufentanil, alfentanil, and remifentanil
  • Diphenylheptanes: Methadone and propoxyphene
  • Benzomorphans: Pentazocine

Commonly Used Opioids

Opiates

Opiates are a class of drugs derived naturally from the opium poppy plant. The psychoactive compounds found in the opium plant include morphine and codeine. Opiates are primarily used for their pain-relieving (analgesic) properties and are often prescribed for the management of moderate-to-severe pain. In addition, they can cause a sense of euphoria and relaxation, which can lead to their misuse and addiction.

Morphine: Morphine is the most widely known and commonly used opioid, often considered the gold standard against which all other opioids are measured. In the acute setting, morphine can be given by intravenous (IV) or oral (PO). Due to its limited bioavailability and first-pass metabolism, the IV-to-PO morphine dosing ratio is approximately 1:3. Morphine can be used for both acute and chronic pain. This drug comes in both immediate-release and extended-release forms. Immediate-release morphine has an onset of 15 to 60 minutes and can last 3 to 7 hours.

For chronic pain management, several extended-release formulations are available, including morphine sulfate, which typically comes in pill formulations 15, 30, 60, 100, and 200 mg. These medications are designed to be taken twice daily and are specifically reserved for patients with chronic pain. Morphine undergoes metabolism, producing an inactive metabolite, morphine-3-glucuronide (M3G), and an active metabolite, morphine-6-glucuronide (M6G), with M3G being the predominant metabolite. Both metabolites are eliminated from the body through urine. 

In renal impairment, these metabolites can accumulate systemically. M6G can cause respiratory depression, and M3G can cause neuroexcitability or seizures. Therefore, morphine as a prolonged treatment in individuals with renal insufficiency is contraindicated. Morphine also causes histamine degranulation from mast cells and can lead to anaphylactoid reactions. IV morphine formulations may contain sulfites, which can lead to allergic reactions in individuals with sulfa allergies.

Codeine: Codeine is a prodrug metabolized to its active form, morphine, in the liver by the cytochrome P450 2D6 enzyme (CYP2D6). Approximately 10% of the population has genetically low levels of CYP2D6; they are poor metabolizers, rendering codeine essentially ineffective.[110] Moreover, varying among ethnicities, between 1% and 11% of individuals are known as CYP2D6 ultra-rapid metabolizers. These individuals metabolize codeine to morphine much faster, increasing the risk of toxic dosing.[111] (A1)

Furthermore, many medications act as inhibitors or inducers of the CYP2D6 system, resulting in issues with polypharmacy. This variability makes codeine a somewhat unpredictable choice of analgesic. Codeine is contraindicated in the pediatric population, and alternatives should also be considered in pregnant and nursing mothers.[112] On average, oral codeine is around one-sixth the strength of oral morphine. Common formulations include codeine mixed with acetaminophen in doses of 15, 30, or 60 mg codeine in combination with 300 mg acetaminophen.

Semi-Synthetic Opioid Derivatives

Semi-synthetic opioid derivatives are opioids that are derived from naturally occurring opiates but are chemically modified in a laboratory setting to enhance their properties or create new drugs. These modifications are typically made to improve the drug's efficacy, potency, or duration of action. Examples of semi-synthetic opioid derivatives include hydrocodone, oxycodone, hydromorphone, and buprenorphine. These drugs are commonly used for pain management but carry a risk of tolerance, dependence, and addiction with long-term use.

Hydrocodone: Hydrocodone is a commonly used opioid only available in PO formulations and has a strength approximately equal to that of PO morphine. This drug is frequently formulated in combination with acetaminophen in doses of 5 to 10 mg hydrocodone with 325 mg acetaminophen. Hydrocodone has an onset of action of 15 to 60 minutes and a duration of 4 to 6 hours. Similar to codeine, hydrocodone is metabolized by the CYP2D6 enzyme; unlike codeine, the parent drug is analgesic and not reliant upon metabolism for efficacy.

Oxycodone: Oxycodone is another opioid available exclusively in oral formulations. This drug has a strength of 1.5 times that of PO morphine. There are a variety of both immediate-release and extended-release oxycodone pills. The immediate release is available in 5, 10, 15, 20, and 30 mg doses. Formulations with acetaminophen include 5 to 10 mg of oxycodone with 325 mg of acetaminophen. The onset of immediate release is 15 to 60 mins, and the duration is 3 to 6 hours. Extended-release doses range from 10 to 80 mg and 9 to 36 mg and have a duration of action of around 12 hours. Extended-release formulations should be reserved for individuals with chronic pain only.

Hydromorphone: Hydromorphone is available in both IV and PO formulations. This drug has a short half-life of about 2.3 hours and is more soluble compared to morphine and far more potent. IV dosing is approximately one-fifth that of morphine, whereas oral dosing is about one-quarter that of PO morphine. PO formulations are available in several doses—1, 2, 4, 8, and 16 mg tablets. Hydromorphone is metabolized by hepatic glucuronidation. Hydromorphone's metabolite hydromorphone-3-glucuronide (H3G) lacks analgesic properties. However, it can cause seizures and other neuroexcitatory effects. Since H3G is excreted through urine, caution must be exercised in patients with kidney disease. 

Buprenorphine: Buprenorphine is a μ-receptor partial agonist that binds with high affinity to the μ-receptor but has weak efficacy at this receptor. This characteristic leads to a ceiling effect, at which higher doses do not produce additional effects, such as respiratory depression. This higher affinity for the μ-receptor can displace pure opioid agonists and induce withdrawal in opioid-dependent individuals. This feature also minimizes the side-effect profile and addiction potential of buprenorphine compared to other pure agonists. Buprenorphine has traditionally been used in combination with naloxone as a treatment for opioid addiction, but given its safer side-effect profile, it is becoming more popular as an analgesic.[113][114](A1)

Buprenorphine has a long half-life of 20 to 70 hours, with a slow onset and long duration of action. Caution should be exhibited when transitioning between other opioids and buprenorphine. The patient's daily opioid dose should be reduced to a maximum of 30 MME/d before implementing treatment with buprenorphine. In general, it is considered that transdermal buprenorphine is 70 to 115 times more potent compared to oral morphine.[113][115][116] (A1)

Buccal buprenorphine has a rough conversion of 1:30 compared to oral morphine. That is, 1 mg of buccal buprenorphine is equal to 30 mg of PO morphine. Formulations include transdermal patches and buccal films. Transdermal patches are available in 5, 10, 20, 35, 52.5, and 70 µg/h patches that last for 1 week. Buccal films are available in 75, 150, 300, 450, 600, 750, and 900 mcg films that can be administered once or twice daily. Buprenorphine can result in QT prolongation.

Synthetic Opioids

Synthetic opioids are a class of opioids that are entirely synthesized in a laboratory rather than being derived from naturally occurring opiates. Examples of synthetic opioids include methadone, meperidine, fentanyl, and tramadol. These drugs are often used for pain management, anesthesia, and sometimes as illicit drugs.

Methadone: Methadone is a synthetic opioid used for both acute and chronic pain and is typically used for the treatment of opioid use disorder due to its long half-life, which can vary from 19 to longer than 45 hours.[117][118] Methadone is increasingly used for chronic pain, particularly neuropathic pain and in patients with opioid tolerance or hyperalgesia, due to its NMDA-antagonist properties. This drug also inhibits serotonin and monoamine reuptake, in addition to its activity on opioid receptors.

Methadone is approximately 70% to 90% bioavailable, and estimations on conversions between methadone and morphine vary significantly from 4 to 12 times the strength of oral morphine. Generally, it is considered non-linear, meaning higher doses of methadone have exponentially increased action compared to lower doses.[119][120][121][122] (A1)

Given its long half-life and potential for accumulation, titration of methadone should be carried out slowly and over an extended period. The timing of methadone-induced respiratory depression can be unpredictable compared to respiratory depression induced by other opioids. Methadone has dose-dependent QT prolongation; therefore, electrocardiogram should be monitored. Methadone is available in 5, 10, and 40 mg tablets, although 40 mg is not approved for chronic pain. This drug is typically administered every 8 hours. Methadone is metabolized by the CYP450 3A4 enzyme and has no active metabolites or extensive renal elimination; therefore, it is not contraindicated for use in patients with renal disease.[123]

Meperidine: Meperidine, one of the first synthetic opioids, is not routinely used for pain in modern practice due to its poor side-effect profile. If used, it is typically reserved for addressing postoperative shivering, as an adjuvant to anesthesia, or for acute pain. This drug is a weak µ-receptor agonist, about one-tenth as potent as morphine, with a faster onset and a shorter duration of action. Meperidine has a notable action in monoamine oxidase inhibition. Furthermore, it has a metabolite, normeperidine, a central nervous system excitant. This metabolite is renally excreted and decreases the seizure threshold.

If taken in combination with other monoamine oxidase inhibitors, it has a significant risk of elucidating serotonin syndrome, making its use in concomitant monoamine oxidase inhibitor usage an absolute contraindication. Meperidine should not be used in pediatric patients and with extreme caution (or avoided) in renal patients. In addition, taking meperidine for extended periods can lead to a build-up of normeperidine; thus, long-term use of meperidine is ill-advised due to the increased likelihood of seizure. Meperidine comes in formulations of 50 and 100 mg tablets.[124] 

Fentanyl: Fentanyl is a potent μ-agonist and an easily titratable IV drug for acute pain. This drug is commonly used for anesthesia due to its rapid onset and short half-life. Fentanyl is metabolized by the CYP3A4 system to inactive metabolites. However, due to its lipophilicity, it is absorbed by the subcutaneous fat, and with prolonged infusions or repeated dosing, fentanyl can have a prolonged context-sensitive half-life. In the chronic pain setting, fentanyl can be used as a transdermal patch.

Transmucosal immediate-release formulations such as nasal sprays, sublingual tablets, oral transmucosal lozenges, buccal tablets, and buccal soluble films also exist. However, patients must be very opioid tolerant before one of these formulations could be considered (≥60 MME/d).

Conversion between other opioids and fentanyl can be difficult due to its complex pharmacodynamics. IV fentanyl to IV morphine exhibits a roughly 1:100 ratio. Fentanyl patches come in 12.5, 25, 50, 75, and 100 μg/hour patches, which can be applied every 3 days and provide steady analgesia throughout this period. A 25 μg/h patch dispenses 600 μg/d, which is equivalent to 60 MME. Fentanyl patches for chronic pain should be used only when other opioids have failed.

Tramadol: Tramadol is a weak but selective μ-opioid receptor agonist and a serotonin and norepinephrine reuptake inhibitor. Given this inhibition of serotonin and norepinephrine, it should be used with caution in individuals taking concomitant antidepressants and can increase the likelihood of seizure and serotonin syndrome. Tramadol is metabolized by CYP2D6 to the active metabolite O-demethyltramadol (M1), which is renally excreted. Caution could be used when prescribing this medication to patients with kidney or liver failure or in older adults. This drug's onset is approximately 1 hour; the peak effect is around 2 to 3 hours, and the half-life is 6 hours. Tramadol has a strength of approximately one-tenth that of PO morphine. Common formulations include 50 and 100 mg instant-release and 100, 150, 200, and 300 mg extended-release.

Differential Diagnosis

Developing a differential diagnosis for chronic pain is based on assessing the possible etiologies of the pain. Identifying the disease process allows clinicians to develop an appropriate treatment plan. For example, a patient's chronic hip and leg pain could manifest as osteoarthritis of the hip, lumbar radiculopathy, or sacroiliac joint dysfunction.

These underlying pathologies may require very different management options. Certain diagnoses may be more responsive to an NSAID compared to a neuropathic pain medication. Clinicians must also consider the possible interventions to address a particular diagnosis—surgical intervention, intra-articular steroid injections, epidural steroid injections, nerve blocks, or radiofrequency ablation procedures.

Clinicians should also consider overlapping diagnoses. For example, patients on chronic opioids may develop opioid-induced hyperalgesia in conjunction with their original presenting pain syndrome. Chronic pain is also frequently accompanied by major depression and sleep disorders.

Acknowledging these coexisting conditions can allow clinicians to choose an appropriate pharmacologic option, such as a serotonin-norepinephrine reuptake inhibitor or tricyclic antidepressant, to address both neuropathic pain and depression symptoms. Furthermore, autoimmune diseases, such as systemic lupus erythematosus or psoriasis, fibromyalgia, and central pain syndromes, should be considered in widespread chronic pain states. Therefore, it is essential to remember that chronic pain can be a symptom of one or multiple diagnoses.

Treatment Planning

Opioid therapy for chronic pain should begin as a trial for a predefined period, typically shorter than 30 days. Treatment goals should be established with the patient before initiating opioid therapy. 

Before initiating opioid therapy, the clinician and patient should agree on a set of practical goals regarding pain relief, functionality with activities of daily living, and management of coexisting conditions such as anxiety and depression. In addition, the plan should include therapy selection, progress measures, and involvement of other specialty healthcare professionals if needed. When developing a pain management plan that involves opioids, the clinician should:

  • Start at the lowest possible dose and titrate to effect
  • Start with short-acting opioid formulations
  • Discuss the need for frequent risk and benefit assessments
  • Educate the patient, and include family members if possible) on the signs and symptoms of respiratory depression
  • Reassess risks and benefits with each dose increase
  • Use extra precaution and be able to justify reason when prescribing 50 MME/d or more to a patient [88]
  • Be knowledgeable of federal and state opioid prescribing regulations
  • Be knowledgeable of patient monitoring, equianalgesic dosing, and incomplete cross-tolerance with opioid conversion
  • Augment treatment with nonopioid medications or, if necessary, immediate-release opioids for breakthrough pain control with long-acting opioids for baseline pain control
  • Taper opioid dose whenever possible

Consent and Treatment

According to the American Medical Association, a physician can proceed with informed consent after assessing the patient's ability to understand the medical information to make an independent decision. The informed consent should include a discussion of the following:

  • Patient diagnosis
  • Purpose of the recommended treatment and its goals
  • Risks and benefits associated with treatment
  • Alternatives to the proposed treatment or no treatment and their associated risks and benefits [125]

Informed Consent for Opioid Therapy Should Include the Following Potential Risks

  • Physical adverse effects or complications
  • Addiction
  • Physical dependence
  • Cognitive effects
  • Psychological dependence
  • Tolerance
  • Hyperalgesia
  • Patient victimization [125] 

Prescribing policies should be clearly described, including policies regarding the number and frequency of refills and procedures for lost or stolen medications.

Patient and Physician Treatment Agreement

There is no consensus on what should be specified in an opioid treatment agreement or contract. However, it should promote a discussion of information, adherence to treatment, and an understanding of treatment goals.[125]

Many clinicians include descriptions in the opioid treatment agreement documentation. These descriptions specify that the patient should agree to use medications safely, use a single prescriber and pharmacy whenever possible to obtain opioid prescriptions, and consent to random drug testing and pill counts. Clinicians also have an active role in the agreement to address issues related to the treatment plan and have appropriate follow-up visits to assess the progress of the interventions.

Reasons for opioid therapy change or discontinuation should be included. Agreements can also include monitoring strategies and mechanisms for the safe storage and disposal of unused drugs. 

Discontinuing Opioid Therapy

Discontinuing opioid therapy should be based on a discussion between physician and patient. Prescribers should consider ending opioid treatment when pain is minimal or resolved, adverse effects develop, analgesia is inadequate despite attempts to optimize opioid therapy, quality of life has not improved, the function has deteriorated, or evidence of aberrant medication use is present. Opioids should be tapered slowly to minimize withdrawal symptoms. An addiction specialist may need to be involved in cases of aberrant behavior or when tapering is difficult to tolerate.

Opioid agonists or antagonists: Opioid agonists or antagonists exhibit variable activity levels on each opioid receptor. Nalbuphine and butorphanol have a high affinity and limited efficacy at the μ-receptor, functioning as antagonists. Nalbuphine is a kappa (κ) agonist, and butorphanol is a partial κ-agonist.[126] Nalbuphine may be utilized to reverse opioid-induced pruritis. Pentazocine is a partial μ-agonist and a κ-agonist.

Opioid partial agonists: Opioid partial agonists have a strong affinity for the μ-receptor, resulting in some efficacy, albeit less than what a complete agonist offers. Buprenorphine is a partial μ-agonist and an κ-antagonist that may be used as an analgesic, a medication-assisted treatment for addiction, or may be formulated with naloxone to deter intravenous abuse. Tramadol is a synthetic 4-phenyl-piperidine codeine analog that acts as a partial μ agonist with activity at the GABA, catecholamine, and serotonin receptors.[127]

Opioid antagonists: Opioid antagonists are competitive μ, κ, and δ receptor antagonists. These antagonists have a strong affinity for μ-opioid receptors but produce no or very low receptor activation, thereby reversing the effects of opioids. Naloxone is a non-selective, competitive opioid receptor antagonist (μ > δ > κ) that acts centrally and peripherally to effectively and quickly reverse opioid overdose due to its rapid onset. However, it also has a rapid offset, with a half-life of approximately 30 to 60 minutes.

As such, when used to treat opioid overdose, naloxone may require re-dosing or an infusion, as the duration of action of the opioid is longer compared to the duration of action of this antidote. There have been reports of resultant pulmonary edema or cardiovascular instability after inducing abrupt opioid withdrawal in patients who are acutely intoxicated or opioid dependent.[128]

Naltrexone also acts centrally and peripherally as a competitive μ-receptor antagonist (μ > κ > δ); it has a long half-life of approximately 24 hours and is typically used to deter abuse. However, it should only be used following opioid detoxification, typically after at least 7 to 10 days without opioid use, to avoid severe withdrawal symptoms.[129] Naloxegol is a peripherally acting opioid antagonist used to treat opioid-induced constipation in select patients.

Pure μ-agonists offer dose-dependent analgesia and side effects. Agonists, antagonists, and partial agonists have a ceiling effect, resulting in a plateau in analgesia and side effects after surpassing a given dose. As a result, these medications offer a safer profile for respiratory depression. For patients using pure opioid agonists, initiating treatment with an agonist, antagonist, or partial agonist may result in withdrawal, as these medications have a higher affinity for the μ-opioid receptor but with less activation of the receptor compared to a pure agonist.

Metabolism of Opioids

Opioids are mainly metabolized by glucuronidation or the liver's CYP450 enzymes, particularly CYP3A4 or 2D6, and then excreted by the kidney. The CYP3A4 enzyme is responsible for the metabolism of more than 50% of all pharmaceuticals; therefore, there is a high risk of drug-drug interactions when using opioids metabolized by the CYP3A4 system. Conversely, the CYP2D6 system metabolizes fewer medications; thus, opioids metabolized by this system have a moderate risk of drug-drug interactions. Opioids metabolized by glucuronidation, such as morphine, hydromorphone, and oxymorphone, have little risk of drug-drug interactions.[130]

The following opioids undergo metabolism by the CYP3A4 system.

  • Fentanyl: After metabolism by CYP3A4, it forms inactive metabolites.
  • Oxycodone: Mostly metabolized by CYP3A4, with a small portion metabolized by CYP2D6 to form oxymorphone (active) and noroxycodone (inactive).
  • Tramadol: Metabolized by both CYP3A4 and CYP2D6, forming its active metabolite O-demethyltramadol (M1), which is eliminated through the kidneys.
  • Methadone: Metabolism involves 6 different CYP450 enzyme systems. Unlike the others, methadone does not produce active metabolites and undergoes complex metabolism without extensive elimination through the kidneys.[130]

The following opioids undergo metabolism by the CYP2D6 system.

  • Hydrocodone: Metabolized by CYP2D6 to its active metabolite hydromorphone.
  • Codeine: After metabolism by CYP2D6, it forms the metabolite morphine.
  • Dihydrocodeine: Metabolized by CYP2D6 to its active metabolite dihydromorphine.
  • Tramadol*: Partially metabolized by CYP2D6.

The CYP2D6 gene exhibits significant genetic variability, leading to a broad spectrum of phenotypes ranging from poor metabolizers to ultrarapid metabolizers.[131] 

Codeine is an inactive pro-drug that must be metabolized to its active form of morphine. As a result, poor metabolizers have poor efficacy with codeine, whereas rapid metabolizers may end up with much higher morphine concentrations much faster, potentially resulting in overdose. Similarly, although tramadol is an active drug, its metabolite M1 has stronger opioid effects compared to tramadol. For this reason, codeine and tramadol are contraindicated for breastfeeding mothers and children younger than 12.[132][133]

The opioids metabolized by phase 2 glucuronidation include morphine, oxymorphone, and hydromorphone. Oxymorphone produces very little active metabolite, less than 1% in total, known as 6-hydroxy-oxymorphone. Morphine is metabolized into 2 main metabolites—M3G (predominantly), which lacks opioid activity and is neuroexcitatory and may cause seizures, and M6G, which offers some analgesia and may contribute to respiratory depression. Hydromorphone is mainly metabolized to H3G, which has neuroexcitatory properties similar to M3G. These metabolites are renally excreted; therefore, caution is advised when using morphine or hydromorphone in older patients or patients with renal failure or dehydration.[130]

Opioid Conversions and Morphine Milligram Equivalents

Potency is the dose required to offer the desired outcome, such as pain control. The equianalgesic dose is the dose at which one opioid at a steady state provides an equal amount of pain relief as another opioid at a steady state.[134] Studies investigating the appropriate opioid conversion ratios from one route to another and from one opioid to another at an equipotent dose are relatively lacking, especially concerning chronic opioid use rather than acute pain management.[135][136]

Conversion ratios vary widely, both for route and conversion from one opioid to another.[134] Furthermore, simple conversion guidance does not consider complexities such as relative half-lives, pharmacologic interactions (for example, certain medications may inhibit the metabolism of particular opioids, leading to prolonged effects), or clinical factors such as liver or renal failure.[134] Prescribers must use their best clinical judgment when transitioning between routes of administration or medication and not rely solely on recommended conversion tables.

When transitioning any given opioid from one route to another, for example, from oral administration to IV administration or vice versa, this rough guidance is generally followed: 

Intrathecal

Epidural

Intravenous

Oral

1

10

100

300

When transitioning from one opioid to another, all potencies are discussed compared to morphine; in other words, all opioid potencies are discussed in terms of their MME. Equianalgesic dose tables represent relative opioid potencies; several such tables exist; examples include the GlobalRPh online calculator and the Agency Medical Directors' Group opioid dose calculator. There is some variability between these equipotent dose recommendations provided by different tables.[137] 

Another consideration to take into account when transitioning from one opioid to another is the concept of incomplete cross-tolerance. Cross-tolerance occurs when there is a reduced effect from a substance after being exposed to a different but pharmacologically similar substance, especially if these substances have action on the same receptor site. With opioids, there is incomplete cross-tolerance.[138] In other words, when transitioning from one opioid to another, the reduction in effect experienced with the new opioid is less compared to what was observed with the previous one. As a result, the new opioid may be used in a lower dose compared to the equipotent dose with a similar effect.

This practice is critical because initiating the new opioid to which the patient is naive at the equipotent dose rather than a somewhat reduced dose could lead to adverse effects such as respiratory depression, for which the patient had developed a tolerance to the previous opioid. Therefore, when switching from one opioid to another, the equipotent dose should typically be decreased by 20% to 50%.[139] In addition, considering relative half-lives is essential when switching from one opioid to another.[134]

When calculating conversions, it is crucial to consider whether the medications involved have similar half-lives. As mentioned above, other considerations must include concurrent drugs and the patient's overall clinical picture. Starting a hypoalgesic dose and titrating up to effect are generally better compared to overshooting and experiencing potentially fatal consequences of respiratory depression.

Importantly, although other opioids are typically converted based on a static MME conversion factor at any given dose, methadone's conversion ratio is dose-dependent; its MME conversion ratio increases as the methadone dose increases.

Toxicity and Adverse Effect Management

The American Society of Interventional Pain Physicians (ASIPP) has established guidelines for prescribing opioids to manage chronic non-cancer pain. To monitor adherence, abuse, and noncompliance, the ASIPP recommends utilizing urine drug tests and PDMPs in addition to clinical and behavioral assessments.[109]

The treatment plan for opioid use in chronic pain should include frequent assessment of pain level, origin, and function. The ASIPP recommends assessing whether pain relief or functional status is improved by at least 30% without adverse effects to continue opioid therapy.[109] If there is a change in dosage or agent, the frequency of patient visits should be increased. Chronic response to opioids should be monitored by evaluating the 5 A's:

  • Affect
  • Aberrant drug-related behaviors
  • Activities of daily living
  • Adverse or side effects
  • Analgesia [140]

Signs and symptoms, if present, suggesting treatment goals are not being achieved include:

  • Decreased appetite
  • Excessive sleeping or day/night reversal
  • Impaired function
  • Inability to concentrate
  • Mood volatility
  • Lack of involvement with others
  • Lack of re-engaging in life
  • Lack of hygiene

The decision to change, continue, or terminate opioid therapy is based on achieving treatment objectives without adverse effects. Therefore, physicians, wherever possible, should work with pharmacists.

Adverse effects of opioids may encompass respiratory depression, constipation, sedation, dizziness, hallucinations, delirium, sedation, falls, immunosuppression, tolerance, physical dependence, sexual dysfunction, osteoporosis, pruritis, nausea or vomiting, and opioid-induced hyperalgesia.

Constipation is one of the most common adverse effects of opioid use. Although tolerance does develop for most opioid adverse effects, it does not develop for constipation. Therefore, clinicians should consider initiating a bowel regimen before the onset of constipation. 

Certain opioids require additional monitoring measures. For example, patients on methadone should have an electrocardiogram periodically to assess for QT interval prolongation.[109]

Acute Overdose Management

Accidental or deliberate overdose is always a risk factor in patients taking opioids. Therefore, the patient and family should be instructed on the signs and symptoms of an overdose and basic emergency management until emergency medical services can arrive.

The immediate response to overdose management is to secure the airway; however, the rapid administration of an opioid antagonist is crucial for survival.[46] Many states allow naloxone distribution to the public. Licensed healthcare professionals may prescribe opioid antagonists for at-risk individuals, relatives, or caregivers. Emergency medical service personnel, police officers, and firefighters can also possess and administer an emergency opioid antagonist. Pharmacists can dispense naloxone without a prescription.

Although opioid antagonists such as naloxone, naltrexone, and nalmefene are available, acute overdoses are typically treated with naloxone as it quickly reverses opioid-related respiratory depression. Naloxone competes with opioids at receptor sites in the brain stem, reversing desensitization to carbon dioxide and preventing respiratory failure. 

For non-healthcare professionals, the naloxone dose for overdose is 0.4 mg administered IM or 2 mg administered intranasally. The dose may be repeated every 2 to 3 minutes, especially for patients who have taken a long-acting or high-dose opioid. Healthcare professionals may administer naloxone through the intravenous, intramuscular, subcutaneous, or intranasal route(s). For emergency medical personnel, patients, and caregivers, naloxone is available in pre-filled auto-injection devices and as a nasal spray. Advanced Cardiac Life Support protocols should be continued while naloxone is being administered.[128][46]

Prognosis

The prognosis associated with acute or chronic pain is highly variable. Prognosis depends upon several factors, including the etiology of the pain or injury, patient-specific factors such as social and genetic predispositions, comorbidities, and many more. Significant predictors of chronic pain prognosis include duration of pain, psychological distress, and age. 

Ultimately, prevention is critical in the treatment of chronic pain. If acute and subacute pain receives appropriate treatment and chronic pain can be avoided, patients have limited impacts on their quality of life. A more extended pain duration history is correlated with increased pain levels, disability, and emotional distress.[141] 

There is an increased association between chronic pain syndromes and psychological comorbidities. The stress of chronic pain contributes to increased emotional stress.

Complications

Chronic pain can significantly reduce the quality of life, productivity, lost wages, worsening comorbidities, and psychiatric disorders, such as depression, anxiety, and substance abuse disorders. Patients with chronic pain also have a significantly increased risk of suicide and suicidal ideation. In addition, clinicians have to consider that many medications used to treat chronic pain have potential complications associated with their use.

Acetaminophen is a common pharmacological therapy for patients with chronic pain. This drug is taken either as a single agent or in combination with other analgesics, including opioids. Acetaminophen-induced hepatotoxicity can occur with doses exceeding 4 g/d.[142] This drug is the most common cause of acute liver failure in the United States.[143] Furthermore, hepatotoxicity can occur at therapeutic doses for patients with chronic liver disease.[144]

Frequently used adjunct medications such as gabapentin or pregabalin can cause sedation, peripheral swelling, mood changes, and confusion. Respiratory depression can occur when these gabapentinoids are used in older patients or patients using opioids and gabapentinoids concurrently.[145]

Feared complications of opioid therapy include addiction and overdose, resulting in respiratory compromise. However, opioid-induced hyperalgesia is also a significant concern. Patients may become more sensitive to painful stimuli while on chronic opioids.[146] Other adverse effects and toxicity management of opioids have been discussed in the preceding sections.

Drug Diversion and Drug Seeking

Unfortunately, due to addiction or financial gain, some individuals seek prescribed opioids for illicit purposes. Prescription opioids may be obtained from a friend or relative, an illicit, a street vendor, a doctor shopping, or through theft from clinical sites.[147] 

Several signs may indicate that a patient is at increased risk or exhibiting behaviors of drug diversion and seeking.[83] These can include:

  • Aggressively demanding more opioids
  • Asking for opioids by name
  • Displaying behaviors suggesting opioid use disorder
  • Forging prescriptions
  • Increasing alcohol use
  • Increasing medication dose without clinician's permission
  • Injecting oral medications
  • Obtaining medications from nonmedical sources
  • Obtaining opioids from multiple clinicians
  • Losing or stealing prescriptions
  • Reluctance to decrease opioid dosing
  • Resisting medication change
  • Requesting early refills
  • Selling prescriptions
  • Sharing or borrowing similar medications
  • Stockpiling medications
  • Using illegal drugs
  • Using pain medications to treat other symptoms

Drug Diversion Interventions

Prescribers and dispensers should implement several precautions to avoid drug diversion. Some approaches include:

  • Communication among clinicians and pharmacies to avoid doctor shopping.
  • Education for patients on the dangers of sharing opioids.
  • Encourage patients to keep opioid medications in a private, locked place.
  • Encourage patients to refrain from public disclosure of opioid use.
  • Report patient prescribing to the state's central database if available.

If a patient is suspected of drug-seeking or diversion, the following actions should be considered:

  • Inquire about prescription and illicit drug use.
  • Obtain a urine drug screen.
  • Perform a thorough examination.
  • Perform pill counting.
  • Prescribe smaller quantities of the opioid.

If a patient is found to be misusing prescribed opioids, this is a violation of the treatment agreement. The clinician may discharge the patient from their practice. If the relationship is terminated, the clinician must do it legally. The clinician should avoid patient abandonment, which means ending a relationship with the patient without considering the continuity of care and without providing notice to the patient. To avoid abandonment charges, the patient must be given enough advanced warning to allow them to secure another clinician and facilitate the transfer of care.

Patients with a substance abuse problem or addiction should be referred to an addiction specialist. Theft or loss of controlled substances should be reported to the Drug Enforcement Administration (DEA). The activity should be documented and reported to law enforcement if drug diversion has occurred.

Consultations

Clinicians involved in long-term chronic pain management should consider consulting with a pain specialist, addiction specialist, or psychiatrist. Clinicians who prescribe opioids should be aware of opioid addiction treatment options.

Deterrence and Patient Education

Involvement of Patient and Family Members

The patient and their family members can assist in making informed decision-making regarding the continuation or cessation of opioid therapy. Family members often notice when a patient experiences depression and decreased functionality. With the patient's consent, clinicians may ask the following questions to family members:

  • Is the patient's day focused on taking opioid pain medication?
  • What is the frequency of pain medication?
  • Does the patient have any other alcohol or drug problems?
  • Does the patient avoid activity?
  • Is the patient depressed?
  • Is the patient able to function?

What To Teach A Patient Taking Opioids

  • Avoid driving or operating power equipment
  • Avoid stopping opioids suddenly
  • Avoid taking other drugs that depress the respiratory system, such as alcohol, sedatives, and anxiolytics
  • Follow up with the prescriber if medication is not adequate for relief
  • Dispose of extra or remaining opioids appropriately
    • DEA-authorized prescription drug take-back programs or collectors (that is, some pharmacies may offer on-site boxes for medication drop-off)
    • Flush the medication down the toilet or dispose of it down the sink drain if the drug is flushable, which can be determined by checking the label, patient information leaflet, or the United States Food and Drug Administration list of drugs that are flushable
    • Mix the medication with coffee grounds, dirt, or cat litter, seal it in an empty can or re-sealable bag or container, and throw it away in the trash [167]
  • Do not share opioids with friends or family
  • Follow the prescribed dosing regimen
  • Provide product-specific information
  • Set appropriate expectations

Interprofessional Approach To Prevent Opioid Abuse or Misuse

Clinicians, pharmacists, and allied healthcare professionals must collaborate as an interprofessional team to ensure the safe and appropriate use of opioid medications. Although clinicians are responsible for initial evaluation and prescription, pharmacists play a crucial role in evaluating the appropriateness and legitimacy of prescriptions received. Failure to assess possible abuse, misuse, or diversion can lead to significant problems for both patients and society. A team approach of prescribers and dispensers likely produces the best outcome.

Pearls and Other Issues

Maintaining Accurate Medical Records for Opioid Prescriptions

All clinicians should maintain accurate, complete, and current medical records, including:

  • Records of controlled substance prescriptions
  • Record instructions provided
  • Detailed history, physical examination, the reason for prescribing, and monitoring measures

Federal and State Laws

Several regulations and programs at the federal and state levels reduce prescription opioid abuse, diversion, and overdose. These laws require:

  • Immunity from prosecution for individuals seeking assistance during an overdose
  • Patient identification before dispensing
  • A physical examination before prescribing opioids
  • Limits and guidelines as they pertain to controlled substance prescriptions
  • Tamper-resistant prescriptions

Federal Laws

The United States DEA sets national standards for controlled substances. Drug scheduling was mandated under The Federal Comprehensive Drug Abuse Prevention and Control Act of 1970, also referred to as the Controlled Substances Act. This act addresses controlled substances within Title II.

The DEA maintains a list of controlled medications and illicit substances categorized from Schedule I to V. The 5 categories are based on the medication's proper and beneficial medical use and the medication's potential for dependency and abuse. The purpose of the law is to provide government oversight over the manufacturing and distribution of these substances. Prescribers and dispensers are required to have a DEA license to supply these drugs. The licensing provides links to users, prescribers, and distributors.[148][149][150][151]

The schedules range from Schedule I to V, with Schedule I drugs having the highest risk of abuse and Schedule V drugs having the lowest potential for abuse. Other factors considered by the DEA include pharmacological effect, evidenced-based knowledge of the drug, risk to public health, trends in the use of the drug, and whether or not the drug has the potential to be modified into a more dangerous drug.

Schedule  
I
  • High abuse potential with no accepted medical use; medications within this schedule may not be prescribed, dispensed, or administered.[149]
  • Examples include marijuana (cannabis), heroin, mescaline (peyote), lysergic acid diethylamide (LSD), methylenedioxymethamphetamine (MDMA), and methaqualone.
II
  • High abuse potential with severe psychological or physical dependence; however, these medications have an accepted medical use and may be prescribed, dispensed, or administered.[149]
  • Examples include fentanyl, oxycodone, morphine, methylphenidate, hydromorphone, amphetamine, methamphetamine (meth), pentobarbital, and secobarbital.
  • Schedule II drugs may not receive a refill.
III
  • Intermediate abuse potential, which is less than Schedule II but more than Schedule IV medications.[149]
  • Examples include products containing 90 mg or less of codeine per dose, buprenorphine, anabolic steroids, testosterone, and ketamine.
IV
  • Abuse potential is less than Schedule III but more than Schedule V medications.[149]
  • Examples include benzodiazepines (including diazepam, alprazolam, midazolam, and lorazepam), carsisoprodol, and tramadol

V
  • Medications with the least potential for abuse among the controlled substances.[149]
  • Examples include cough medications containing ≤200 mg of codeine per 100 mL or per 100 g, pregabalin, diphenoxylate or atropine, and dextromethorphan.

Understanding the DEA controlled-substance scheduling is essential to ensure adequate caution when prescribing medications with high abuse potential and to ensure against prescribing outside of one's authority.[152][153]

The Controlled Substances Act aims to improve patient safety by providing federal oversight for drugs with potential for abuse. Healthcare professionals of scheduled substance prescriptions, including physicians, dentists, podiatrists, and advanced practitioners, must have a DEA license and record prescriptions of scheduled drugs. The goals are to reduce overprescribing and to help identify potential drug-seeking patients.[154][155]

The dispenser must also be aware of a patient's medication history and be mindful of the potential for polypharmacy if a patient seeks multiple healthcare professionals. Given the current opioid epidemic, federal oversight and interdisciplinary coordination strive to reduce harm to patients prescribed scheduled drugs. However, further research and time are needed to assess whether drug scheduling reduces abuse, addiction, and overdose.[156][157][158]

Florida State Standards of Practice for Treatment of Chronic Nonmalignant Pain (Florida State Statute 456.4-Controlled Substance Prescribing)

These standards of practice apply to physicians licensed under chapters 458, 459, 461, or 466, physician assistants licensed under chapters 458 or 459, or advanced practice registered nurses licensed under 464 (part I) who prescribe any controlled substance listed in Schedule II, Schedule III, or Schedule IV for the treatment of chronic nonmalignant pain.

  • A detailed medical history and a thorough physical examination must be completed before beginning any treatment
  • The medical record must include:
    • The nature and intensity of the pain and the effect of pain on physical and psychosocial functioning
    • Current and past treatments
    • Coexisting diseases or conditions and a review of previous medical records
    • History of alcohol and substance abuse, if present
  • A written plan for assessing potential aberrant drug-related behavior must be formulated and should include patient drug testing.
  • The risks and benefits of controlled substance therapy must be discussed with the patient, highlighting the potential for abuse, addiction, and physical dependence.
  • A written controlled substance agreement between the registrant and the patient outlining the patient's responsibilities should be maintained.
  • Patient compliance with medication, treatment plan, and agreement responsibilities should be monitored at least every 3 months.
  • The use of controlled substances in patients with a history of substance abuse or a comorbid psychiatric disorder requires consultation with an addiction medicine specialist or a psychiatrist.
  • Patients with signs or symptoms of substance abuse must immediately be referred to a board-certified pain management physician, an addiction medicine specialist, or a mental health addiction facility.
  • Evidence of diversion should be followed by discontinuation of controlled substance therapy.

Florida State Standards of Practice for Treatment of Acute Pain (Florida State Statute 456.4-Controlled Substance Prescribing)

  • Controlled substance prescription for acute pain may not exceed a 3-day supply.
  • A 7-day supply for acute pain may be prescribed if the prescriber believes that more than a 3-day supply is medically necessary and indicates acute pain exception on the script.
    • The prescriber must also document the acute medical condition and lack of alternative treatment options to justify deviation from the 3-day supply limit.
  • The prescriber must concurrently prescribe an emergency opioid antagonist.
  • Information on available nonopioid alternatives for pain treatment must be provided to the patient to ensure an informed decision is made.
  • The prescriber must discuss the advantages and disadvantages of nonopioid therapies with the patient or their representative.

12 Core Competencies for Prescribing Controlled Substances

  • Understand the epidemiology and population level of opioid addiction.
  • Develop competence in performing a pain evaluation.
  • Implement both pharmacological and nonpharmacological approaches to pain management.
  • Understand the practical aspects of prescribing and communication.
  • Develop techniques for conflict prevention and resolution.
  • Adhere to established goals for managing chronic pain.
  • Understand acute pain care for chronic pain patients.
  • Develop an interoffice and interprofessional focus.
  • Conduct thorough assessments for substance use disorder risk.
  • Develop the treatment plan for patients with substance use disorder.
  • Educate patients and their family members on the management of overdose risk.
  • Understand the professional and legal standards.

Enhancing Healthcare Team Outcomes

Chronic pain affects millions of individuals and is a significant public health concern with considerable morbidity and mortality, especially if associated with opioid diversion and misuse. Without proper management, the patient's quality of life can be deleterious. Thus, chronic pain is best managed with an interprofessional approach.

Chronic pain correlates with several severe complications, including severe depression and suicidal ideation and attempts. The lifetime prevalence of chronic pain patients attempting suicide was shown to be between 5% and 14%; suicidal ideation was approximately 20%.[37] 

These complications often require psychiatric intervention and advanced pharmacological or interventional therapies. Severe symptoms must be treated immediately, leading to an increase in healthcare expenses. Identifying the risk factors, performing a thorough assessment of the patient with chronic pain, and monitoring the progression of symptoms are all important. A team approach is an ideal way to limit the effects of chronic pain and its complications through the following recommendations: 

  • Diagnosis of a patient with acute pain by the primary care provider to prevent the progression of chronic pain is the recommended first step.
  • Conservative chronic pain management should begin with mild or moderate symptoms, including physical therapy, cognitive behavioral therapy, and pharmacological management.
  • A pharmacist or an experienced clinician should review the medication regimen, including medication reconciliation, to preclude drug-drug interactions and alert the healthcare team regarding any concerns.
  • Regular follow-up with a primary care provider and pain specialist is crucial for assessing and effectively managing the patient's pain.
  • Clinicians must address comorbid psychiatric disorders. Depending on the severity of the patient's symptoms, this action may require a psychiatrist's involvement.
  • If symptoms worsen on follow-up or there is a concerning escalation of pharmacological therapy, such as with opioids, a referral to a pain medicine specialist merits consideration.
  • If the patient has exhausted various pharmacological and nonpharmacological treatment options, interventional procedures may be considered.
  • If the patient expresses any concerns about suicidal thoughts or plans at any point, an emergent psychiatric team should examine the patient immediately.
  • Patients who have developed opioid dependence secondary to pharmacological therapy should be offered treatment and possible referral for addiction treatment or detoxification if indicated. The patient should be put on a medication-weaning schedule or should be provided with medications to treat opioid dependence.
  • Based on CDC recommendations, patients on high-dose opioid medications or with risk factors for opioid overdose, such as obesity, sleep apnea, concurrent benzodiazepine use, and high daily opioid dosages, should receive naloxone at home for the emergent treatment of an unintentional overdose.[159]

Effective and open interprofessional communication is crucial for the optimal management of chronic pain and for minimizing its adverse effects on the patient.

Interprofessional Team Case Study #1

Chief complaint: Syncope and confusion

History of present illness: A 70-year-old woman is brought to the hospital after being unconscious on the bathroom floor. Her husband reports a history of morbid obesity, diabetes, kidney disease, and chronic right hip pain. Hip replacement surgery has been delayed due to difficulties in controlling her weight. She has been prescribed a combination of hydrocodone-acetaminophen for pain control, enabling her to ambulate and perform daily pool exercises. Lately, her pain has worsened, and her husband mentions that she occasionally takes more medicine than prescribed.

Physical examination: Vital signs include a temperature of 98.7 °F (37.0 °C), blood pressure of 100/70 mm Hg, heart rate of 110 bpm, respiratory rate of 14 breaths/min, and O2 saturation of 92%.

A physical examination reveals that the patient is confused but somnolent, with arousal only to painful stimuli. Cardiac and pulmonary examinations are normal. There are no focal neurologic deficits.

Workup: Laboratory studies, including a complete blood cell count, chemistry panel, and urinalysis, are normal except for a moderate elevation of blood urea nitrogen and creatinine. Chest and pelvic radiographs are negative for fracture. The electrocardiogram was normal. The urine drug screen is positive for opioids and cannabinoids.

The patient is given naloxone and quickly regains consciousness. She reports that the last thing she remembers is feeling faint while urinating. She indicates that she has difficulty sleeping and walking due to chronic hip pain, and she has severe pain with exercise in their home pool. She denies suicidal ideation. She admits to taking an extra pain pill and smoking a joint before the episode.

A PDMP inquiry reveals that her orthopedic surgeon has prescribed her 3 times her daily dose of hydrocodone-acetaminophen for the last 6 weeks.

Treatment: The patient is admitted to the hospital, and orthopedic surgery, pain management, pharmacy, physical therapy, and dietician are consulted. A subsequent workup by the interprofessional team concluded that the incident was caused by an accidental overdose of her opioid medication combined with her comorbid conditions.

The patient's family and husband were told that because of the patient's age, diabetes, and kidney disease, her body processes the opioid medication less effectively. With the doubling of her dose, she developed acute toxic encephalopathy. The patient and family have a limited understanding of the potential adverse effects of opioids in treating pain and poor performance in exercising and dieting for weight control.

The interprofessional team recommends opioids at the prescribed dose without additional doses, NSAIDs sparingly for breakthrough pain, monitored physical therapy and exercise, a planned diet, use of a transcutaneous electrical nerve stimulation unit, and follow-up with a pain specialist and orthopedic surgeon for surgical intervention as soon as possible. 

At discharge, a written treatment and management plan is presented to the patient and family for discussion and consent. The goals include pain control, increased exercise, weight loss, and surgical intervention for hip replacement.

The patient is scheduled for outpatient pain management, physical therapy, and dietician appointments. Before discharge, the pharmacist counsels the patient and family regarding the safe use, dosage regulation, adverse effects, and proper disposal of opioid medication. Naloxone is prescribed, and the family is educated on its use.

Follow-up: Three months after discharge, the patient had successfully lost 40 pounds (18 kg) and received clearance for surgery. Following hip replacement, the patient's pain has dramatically diminished, and she has gradually tapered off her opioid medications. She continues to maintain regular exercise and diet. She reports that she feels better, enjoys life, and is encouraged by her progress.

An interprofessional approach to diagnosis, treatment, and follow-up provides a positive outcome.

Interprofessional Case Study #2

Chief complaint: Request For increased pain medicine

A 76-year-old woman presents to the pharmacy to refill a prescription for acetaminophen-codeine. Upon inquiry, the pharmacist learns that the patient has been taking this medication for several months after breaking her back when she tripped over her hungry cat. The patient mentions that she recently relocated to live with her daughter, who resides nearby.

The pharmacist further inquires about any issues the patient may be experiencing, and she states that she has the runs, sleeps as much as her cat, and hurts all over. She expresses a desire to resume her volunteer work as a librarian but finds it challenging due to her ongoing pain, which limits her ability to bend and carry out her duties effectively. The pharmacist observes that the patient appears slightly agitated.

The pharmacist accesses PDMP and confirms that acetaminophen-codeine is the only narcotic the patient has been prescribed for the last 6 months, with a recent drop in frequency from 4 to 2 times daily dosing. The pharmacist is concerned about chronic pain, and long-term treatment with an opioid is a risk factor for opioid use disorder.

The pharmacist contacts the orthopedic surgeon who prescribed the medication and presents concerns regarding the development of opioid use disorder. After presenting the pharmacist's observations, the clinician agrees to reassess the patient's treatment plan. As a result, the dosage is adjusted to 3 times daily, and the clinician schedules an appointment to meet with the patient in the office the following day.

Workup or Treatment: During the visit, the orthopedic surgeon conducts a follow-up x-ray and confirms that the lumbar vertebral fracture has healed. In addition to maintaining the current 3-times-daily opioid dosage, the surgeon adds a low-dose NSAID for another 30 days and recommends physical therapy for the patient. In addition, the surgeon instructs the patient to apply hot and cold ice packs to her lower back, get a weekly massage, and start gentle stretching exercises. The patient is instructed to follow up with the clinician's nurse practitioner in 2 weeks for further evaluation and management.

Follow-up: The patient sees the nurse practitioner after 14 days. Her pain is under control, and the signs and symptoms of opioid use disorder have resolved. The patient is instructed to continue the opioid twice daily and is placed on a long-term NSAID with a higher safety profile. She is instructed to return in 14 days.

The orthopedic surgeon sees the patient on the next visit and reviews the physical therapist's report. The patient feels much better, and the opioid is decreased to once daily for 14 days. Then, she is instructed to stop the medication and return for a follow-up visit 2 days later.

The patient reports minimal pain on this visit and has no symptoms of opioid withdrawal or use disorder. She is kept on the NSAID at a once-daily dose and continues to see the physical therapist for another month.

During her follow-up with the nurse practitioner, a low-dose NSAID with a high safety profile is prescribed. She sees the pharmacist who refills the NSAID and reports that she is back to work at the library. She is feeling much better and now feeds the cat on the counter to avoid tripping over it during feeding.

An interprofessional approach to diagnosis, treatment, and dispensing provides a positive outcome.

Interprofessional Case Study #3

Chief complaint: Chronic leg pain

History of present illness: A 60-year-old man presents with a history of progressive neuropathic leg pain with an unclear etiology. Despite trying physical therapy, relaxation therapy, massage therapy, and acupuncture, he has found no relief. Gabapentin, selective serotonin reuptake inhibitors, NSAIDs, and tramadol controlled the pain for the last year, but the pain is now worse and inhibits the patient's mobility. In the last 2 weeks, the clinician prescribed a short-acting opioid, which provided relief.

Physical examination: Vital signs include a temperature of 98.8 °F (37.1 °C), blood pressure of 120/80 mm Hg, heart rate of 90 bpm, respiratory rate of 16 breaths/min, and O2 saturation of 98%.

A physical examination reveals a male in no apparent distress. The heart, lung, and extremity examination is normal. A neurological examination revealed hyperalgesia and hyperpathia of the lower legs.

Workup: Prior electromyography studies were consistent with severe neuropathy. Laboratory blood tests reveal no vitamin deficits or toxicities.

Treatment: The neurologist decides to transition the patient from short-acting to long-acting opioids and discusses the risks and benefits. The nurse provides a hand-out and discusses topics such as monitoring, understanding, signing required documentation, safe opioid use, safe storage, expectations, addiction risks, and required regular follow-ups. The nurse also discusses the signs and symptoms of opioid overdose and the use of naloxone.

The clinicians prescribe a long-acting narcotic at a low dose and naloxone. The patient is asked to follow up in 7 days.

The pharmacist is familiar with the patient. The pharmacist accesses PDMP and confirms the prior narcotic order, the new long-acting order, and that no other potentially interactive medications have been prescribed. The pharmacist educates the patient on narcotic and naloxone use.

Follow-up: The patient returned for a follow-up appointment at 7 days, and the pain was under control with no complications. The prescriber completed appropriate documentation and provided a 30-day course of treatment with monthly follow-ups.

References


[1]

Nahin RL. Estimates of pain prevalence and severity in adults: United States, 2012. The journal of pain. 2015 Aug:16(8):769-80. doi: 10.1016/j.jpain.2015.05.002. Epub 2015 May 29     [PubMed PMID: 26028573]


[2]

Imtiaz S, Shield KD, Fischer B, Elton-Marshall T, Sornpaisarn B, Probst C, Rehm J. Recent changes in trends of opioid overdose deaths in North America. Substance abuse treatment, prevention, and policy. 2020 Aug 31:15(1):66. doi: 10.1186/s13011-020-00308-z. Epub 2020 Aug 31     [PubMed PMID: 32867799]


[3]

Kedia S, Ahuja N, Wyant DK, Dillon PJ, Akkus C, Relyea G. Compositional and contextual factors associated with drug overdose deaths in the United States. Journal of addictive diseases. 2020 Apr-Jun:38(2):143-152. doi: 10.1080/10550887.2020.1729079. Epub 2020 Mar 20     [PubMed PMID: 32195626]


[4]

Chang HY, Kharrazi H, Bodycombe D, Weiner JP, Alexander GC. Healthcare costs and utilization associated with high-risk prescription opioid use: a retrospective cohort study. BMC medicine. 2018 May 16:16(1):69. doi: 10.1186/s12916-018-1058-y. Epub 2018 May 16     [PubMed PMID: 29764482]

Level 2 (mid-level) evidence

[5]

Alford DP, Krebs EE, Chen IA, Nicolaidis C, Bair MJ, Liebschutz J. Update in pain medicine. Journal of general internal medicine. 2010 Nov:25(11):1222-6. doi: 10.1007/s11606-010-1452-4. Epub 2010 Jul 15     [PubMed PMID: 20632120]


[6]

Pizzo PA, Clark NM. Alleviating suffering 101--pain relief in the United States. The New England journal of medicine. 2012 Jan 19:366(3):197-9. doi: 10.1056/NEJMp1109084. Epub     [PubMed PMID: 22256802]


[7]

Treede RD, Rief W, Barke A, Aziz Q, Bennett MI, Benoliel R, Cohen M, Evers S, Finnerup NB, First MB, Giamberardino MA, Kaasa S, Korwisi B, Kosek E, Lavand'homme P, Nicholas M, Perrot S, Scholz J, Schug S, Smith BH, Svensson P, Vlaeyen JWS, Wang SJ. Chronic pain as a symptom or a disease: the IASP Classification of Chronic Pain for the International Classification of Diseases (ICD-11). Pain. 2019 Jan:160(1):19-27. doi: 10.1097/j.pain.0000000000001384. Epub     [PubMed PMID: 30586067]


[8]

Ruiz Romero MV, Lobato Parra E, Porrúa Del Saz A, Martínez Monrobé MB, Pereira Delgado CM, Gómez Hernández MB. [Management of chronic non-oncologic pain by multicomponent programs using non-pharmacologic therapies: A systematic review of the literature]. Journal of healthcare quality research. 2024 May-Jun:39(3):168-187. doi: 10.1016/j.jhqr.2024.02.004. Epub 2024 Mar 30     [PubMed PMID: 38556371]

Level 1 (high-level) evidence

[9]

Balit J, Erlangsen A, Docherty A, Turecki G, Orri M. Association of chronic pain with suicide attempt and death by suicide: a two-sample Mendelian randomization. Molecular psychiatry. 2024 Feb 16:():. doi: 10.1038/s41380-024-02465-0. Epub 2024 Feb 16     [PubMed PMID: 38366113]


[10]

Dydyk AM, Conermann T. Chronic Pain. StatPearls. 2024 Jan:():     [PubMed PMID: 31971706]


[11]

Nunes JC, Costa GPA, Weleff J, Rogan M, Compton P, De Aquino JP. Assessing pain in persons with opioid use disorder: Approaches, techniques and special considerations. British journal of clinical pharmacology. 2024 Mar 31:():. doi: 10.1111/bcp.16055. Epub 2024 Mar 31     [PubMed PMID: 38556851]


[12]

Shipton EE, Bate F, Garrick R, Steketee C, Shipton EA, Visser EJ. Systematic Review of Pain Medicine Content, Teaching, and Assessment in Medical School Curricula Internationally. Pain and therapy. 2018 Dec:7(2):139-161. doi: 10.1007/s40122-018-0103-z. Epub 2018 Jul 30     [PubMed PMID: 30058045]

Level 1 (high-level) evidence

[13]

Wahezi SE, Emerick TD, Caparó M, Choi H, Eshraghi Y, Naeimi T, Kohan L, Anitescu M, Wright T, Przkora R, Patel K, Lamer TJ, Moeschler S, Yener U, Alerte J, Grandhe R, Bautista A, Spektor B, Noon K, Reddy R, Osuagwu UC, Carpenter A, Gerges FJ, Horn DB, Murphy CA, Kim C, Pritzlaff SG, Marshall C, Kirchen G, Oryhan C, Swaran Singh TS, Sayed D, Lubenow TR, Sehgal N, Argoff CE, Gulati A, Day MR, Shaparin N, Sibai N, Dua A, Barad M. The current state of training in pain medicine fellowships: An Association of Pain Program Directors (APPD) survey of program directors. Pain practice : the official journal of World Institute of Pain. 2024 Mar 30:():. doi: 10.1111/papr.13373. Epub 2024 Mar 30     [PubMed PMID: 38553945]

Level 3 (low-level) evidence

[14]

Yorkgitis BK, Raygor D, Bryant E, Brat G, Smink DS, Crandall M. Surgery program directors' knowledge of opioid prescribing regulations: a survey study. The Journal of surgical research. 2018 Jul:227():194-197. doi: 10.1016/j.jss.2018.02.036. Epub 2018 Mar 20     [PubMed PMID: 29804853]

Level 3 (low-level) evidence

[15]

Dydyk AM, Sizemore DC, Fariba KA, Sanghavi DK, Porter BR. Florida Controlled Substance Prescribing. StatPearls. 2024 Jan:():     [PubMed PMID: 33428370]


[16]

Dydyk AM, Sizemore DC, Haddad LM, Lindsay L, Porter BR. NP Safe Prescribing of Controlled Substances While Avoiding Drug Diversion. StatPearls. 2024 Jan:():     [PubMed PMID: 33232099]


[17]

Singier A, Noize P, Berdaï D, Daveluy A, Arnaud M, Molimard M, Bégaud B, Salvo F. Medicine misuse: A systematic review and proposed hierarchical terminology. British journal of clinical pharmacology. 2021 Apr:87(4):1695-1704. doi: 10.1111/bcp.14604. Epub 2020 Dec 8     [PubMed PMID: 33295072]

Level 1 (high-level) evidence

[18]

Alenezi A, Yahyouche A, Paudyal V. Interventions to optimize prescribed medicines and reduce their misuse in chronic non-malignant pain: a systematic review. European journal of clinical pharmacology. 2021 Apr:77(4):467-490. doi: 10.1007/s00228-020-03026-4. Epub 2020 Oct 30     [PubMed PMID: 33123784]

Level 1 (high-level) evidence

[19]

Karmali RN, Bush C, Raman SR, Campbell CI, Skinner AC, Roberts AW. Long-term opioid therapy definitions and predictors: A systematic review. Pharmacoepidemiology and drug safety. 2020 Mar:29(3):252-269. doi: 10.1002/pds.4929. Epub 2019 Dec 18     [PubMed PMID: 31851773]

Level 1 (high-level) evidence

[20]

Yasaei R, Peterson E, Saadabadi A. Chronic Pain Syndrome. StatPearls. 2024 Jan:():     [PubMed PMID: 29262143]


[21]

Hardt J, Jacobsen C, Goldberg J, Nickel R, Buchwald D. Prevalence of chronic pain in a representative sample in the United States. Pain medicine (Malden, Mass.). 2008 Oct:9(7):803-12. doi: 10.1111/j.1526-4637.2008.00425.x. Epub 2008 Mar 11     [PubMed PMID: 18346058]


[22]

Annagür BB, Uguz F, Apiliogullari S, Kara I, Gunduz S. Psychiatric disorders and association with quality of sleep and quality of life in patients with chronic pain: a SCID-based study. Pain medicine (Malden, Mass.). 2014 May:15(5):772-81. doi: 10.1111/pme.12390. Epub 2014 Feb 25     [PubMed PMID: 24612225]

Level 2 (mid-level) evidence

[23]

Singh GK, Kim IE, Girmay M, Perry C, Daus GP, Vedamuthu IP, De Los Reyes AA, Ramey CT, Martin EK, Allender M. Opioid Epidemic in the United States: Empirical Trends, and A Literature Review of Social Determinants and Epidemiological, Pain Management, and Treatment Patterns. International journal of MCH and AIDS. 2019:8(2):89-100. doi: 10.21106/ijma.284. Epub 2019 Aug 8     [PubMed PMID: 31723479]

Level 2 (mid-level) evidence

[24]

Cerdá M, Krawczyk N, Hamilton L, Rudolph KE, Friedman SR, Keyes KM. A Critical Review of the Social and Behavioral Contributions to the Overdose Epidemic. Annual review of public health. 2021 Apr 1:42():95-114. doi: 10.1146/annurev-publhealth-090419-102727. Epub 2021 Nov 30     [PubMed PMID: 33256535]


[25]

Rigg KK, March SJ, Inciardi JA. Prescription Drug Abuse & Diversion: Role of the Pain Clinic. Journal of drug issues. 2010:40(3):681-702     [PubMed PMID: 21278927]


[26]

Harvey LM, Fan W, Cano MÁ, Vaughan EL, Arbona C, Essa S, Sanchez H, de Dios MA. Psychosocial intervention utilization and substance abuse treatment outcomes in a multisite sample of individuals who use opioids. Journal of substance abuse treatment. 2020 May:112():68-75. doi: 10.1016/j.jsat.2020.01.016. Epub 2020 Jan 25     [PubMed PMID: 32199548]


[27]

Han Y, Yan W, Zheng Y, Khan MZ, Yuan K, Lu L. The rising crisis of illicit fentanyl use, overdose, and potential therapeutic strategies. Translational psychiatry. 2019 Nov 11:9(1):282. doi: 10.1038/s41398-019-0625-0. Epub 2019 Nov 11     [PubMed PMID: 31712552]


[28]

Ruhm CJ. Drivers of the fatal drug epidemic. Journal of health economics. 2019 Mar:64():25-42. doi: 10.1016/j.jhealeco.2019.01.001. Epub 2019 Jan 14     [PubMed PMID: 30784811]


[29]

Warp PV, Hauschild M, Serota DP, Ciraldo K, Cruz I, Bartholomew TS, Tookes HE. A confirmed case of xylazine-induced skin ulcers in a person who injects drugs in Miami, Florida, USA. Harm reduction journal. 2024 Mar 15:21(1):64. doi: 10.1186/s12954-024-00978-z. Epub 2024 Mar 15     [PubMed PMID: 38491467]

Level 3 (low-level) evidence

[30]

Haffajee RL, Mello MM. Drug Companies' Liability for the Opioid Epidemic. The New England journal of medicine. 2017 Dec 14:377(24):2301-2305. doi: 10.1056/NEJMp1710756. Epub     [PubMed PMID: 29236640]


[31]

Davis CS, Carr DH. Self-regulating profession? Administrative discipline of "pill mill" physicians in Florida. Substance abuse. 2017 Jul-Sep:38(3):265-268. doi: 10.1080/08897077.2017.1316812. Epub 2017 Apr 10     [PubMed PMID: 28394740]


[32]

Eukel H, Steig J, Frenzel O, Skoy E, Werremeyer A, Strand M. Opioid Misuse and Overdose: Changes in Pharmacist Practices and Outcomes. The Journal of continuing education in the health professions. 2020 Fall:40(4):242-247. doi: 10.1097/CEH.0000000000000317. Epub     [PubMed PMID: 33284175]


[33]

Hoppe J, Howland MA, Nelson L. The role of pharmacies and pharmacists in managing controlled substance dispensing. Pain medicine (Malden, Mass.). 2014 Dec:15(12):1996-8. doi: 10.1111/pme.12531. Epub 2014 Aug 19     [PubMed PMID: 25138390]


[34]

Tanabe P, Buschmann M. A prospective study of ED pain management practices and the patient's perspective. Journal of emergency nursing. 1999 Jun:25(3):171-7     [PubMed PMID: 10346837]

Level 2 (mid-level) evidence

[35]

Downey LV, Zun LS. Pain management in the emergency department and its relationship to patient satisfaction. Journal of emergencies, trauma, and shock. 2010 Oct:3(4):326-30. doi: 10.4103/0974-2700.70749. Epub     [PubMed PMID: 21063553]


[36]

Dahlhamer J, Lucas J, Zelaya C, Nahin R, Mackey S, DeBar L, Kerns R, Von Korff M, Porter L, Helmick C. Prevalence of Chronic Pain and High-Impact Chronic Pain Among Adults - United States, 2016. MMWR. Morbidity and mortality weekly report. 2018 Sep 14:67(36):1001-1006. doi: 10.15585/mmwr.mm6736a2. Epub 2018 Sep 14     [PubMed PMID: 30212442]


[37]

Tang NK, Crane C. Suicidality in chronic pain: a review of the prevalence, risk factors and psychological links. Psychological medicine. 2006 May:36(5):575-86     [PubMed PMID: 16420727]


[38]

Petrosky E, Harpaz R, Fowler KA, Bohm MK, Helmick CG, Yuan K, Betz CJ. Chronic Pain Among Suicide Decedents, 2003 to 2014: Findings From the National Violent Death Reporting System. Annals of internal medicine. 2018 Oct 2:169(7):448-455. doi: 10.7326/M18-0830. Epub 2018 Sep 11     [PubMed PMID: 30208405]


[39]

Feldt KS, Ryden MB, Miles S. Treatment of pain in cognitively impaired compared with cognitively intact older patients with hip-fracture. Journal of the American Geriatrics Society. 1998 Sep:46(9):1079-85     [PubMed PMID: 9736099]


[40]

Scher C, Meador L, Van Cleave JH, Reid MC. Moving Beyond Pain as the Fifth Vital Sign and Patient Satisfaction Scores to Improve Pain Care in the 21st Century. Pain management nursing : official journal of the American Society of Pain Management Nurses. 2018 Apr:19(2):125-129. doi: 10.1016/j.pmn.2017.10.010. Epub 2017 Dec 15     [PubMed PMID: 29249620]


[41]

Van Zee A. The promotion and marketing of oxycontin: commercial triumph, public health tragedy. American journal of public health. 2009 Feb:99(2):221-7. doi: 10.2105/AJPH.2007.131714. Epub 2008 Sep 17     [PubMed PMID: 18799767]


[42]

Cicero TJ, Inciardi JA, Muñoz A. Trends in abuse of Oxycontin and other opioid analgesics in the United States: 2002-2004. The journal of pain. 2005 Oct:6(10):662-72     [PubMed PMID: 16202959]

Level 2 (mid-level) evidence

[43]

Manchikanti L, Singh A. Therapeutic opioids: a ten-year perspective on the complexities and complications of the escalating use, abuse, and nonmedical use of opioids. Pain physician. 2008 Mar:11(2 Suppl):S63-88     [PubMed PMID: 18443641]

Level 3 (low-level) evidence

[44]

Wang J, Christo PJ. The influence of prescription monitoring programs on chronic pain management. Pain physician. 2009 May-Jun:12(3):507-15     [PubMed PMID: 19461820]


[45]

Grewal N, Huecker MR. Opioid Prescribing. StatPearls. 2024 Jan:():     [PubMed PMID: 31869184]


[46]

Schiller EY, Goyal A, Mechanic OJ. Opioid Overdose. StatPearls. 2024 Jan:():     [PubMed PMID: 29262202]


[47]

Smith WR, Valrie CR, Jaja C, Kenney MO. Precision, integrative medicine for pain management in sickle cell disease. Frontiers in pain research (Lausanne, Switzerland). 2023:4():1279361. doi: 10.3389/fpain.2023.1279361. Epub 2023 Nov 9     [PubMed PMID: 38028431]


[48]

Nicholas M, Vlaeyen JWS, Rief W, Barke A, Aziz Q, Benoliel R, Cohen M, Evers S, Giamberardino MA, Goebel A, Korwisi B, Perrot S, Svensson P, Wang SJ, Treede RD, IASP Taskforce for the Classification of Chronic Pain. The IASP classification of chronic pain for ICD-11: chronic primary pain. Pain. 2019 Jan:160(1):28-37. doi: 10.1097/j.pain.0000000000001390. Epub     [PubMed PMID: 30586068]


[49]

Rayner L, Hotopf M, Petkova H, Matcham F, Simpson A, McCracken LM. Depression in patients with chronic pain attending a specialised pain treatment centre: prevalence and impact on health care costs. Pain. 2016 Jul:157(7):1472-9. doi: 10.1097/j.pain.0000000000000542. Epub     [PubMed PMID: 26963849]


[50]

Meyer-Rosberg K, Kvarnström A, Kinnman E, Gordh T, Nordfors LO, Kristofferson A. Peripheral neuropathic pain--a multidimensional burden for patients. European journal of pain (London, England). 2001:5(4):379-89     [PubMed PMID: 11743704]


[51]

Compton WM, Jones CM. Epidemiology of the U.S. opioid crisis: the importance of the vector. Annals of the New York Academy of Sciences. 2019 Sep:1451(1):130-143. doi: 10.1111/nyas.14209. Epub 2019 Aug 4     [PubMed PMID: 31378974]


[52]

LaForge KS, Yuferov V, Kreek MJ. Opioid receptor and peptide gene polymorphisms: potential implications for addictions. European journal of pharmacology. 2000 Dec 27:410(2-3):249-268     [PubMed PMID: 11134674]

Level 3 (low-level) evidence

[53]

Kreek MJ, Bart G, Lilly C, LaForge KS, Nielsen DA. Pharmacogenetics and human molecular genetics of opiate and cocaine addictions and their treatments. Pharmacological reviews. 2005 Mar:57(1):1-26     [PubMed PMID: 15734726]


[54]

Trescot AM, Datta S, Lee M, Hansen H. Opioid pharmacology. Pain physician. 2008 Mar:11(2 Suppl):S133-53     [PubMed PMID: 18443637]


[55]

Harris HN, Peng YB. Evidence and explanation for the involvement of the nucleus accumbens in pain processing. Neural regeneration research. 2020 Apr:15(4):597-605. doi: 10.4103/1673-5374.266909. Epub     [PubMed PMID: 31638081]


[56]

Wolff K, Rostami-Hodjegan A, Shires S, Hay AW, Feely M, Calvert R, Raistrick D, Tucker GT. The pharmacokinetics of methadone in healthy subjects and opiate users. British journal of clinical pharmacology. 1997 Oct:44(4):325-34     [PubMed PMID: 9354306]


[57]

Oelhaf RC, Del Pozo E, Azadfard M. Opioid Toxicity. StatPearls. 2024 Jan:():     [PubMed PMID: 28613731]


[58]

Kaye AD, Dufrene K, Cooley J, Walker M, Shah S, Hollander A, Shekoohi S, Robinson CL. Neuropsychiatric Effects Associated with Opioid-Based Management for Palliative Care Patients. Current pain and headache reports. 2024 Apr 2:():. doi: 10.1007/s11916-024-01248-0. Epub 2024 Apr 2     [PubMed PMID: 38564124]


[59]

Polomano RC, Galloway KT, Kent ML, Brandon-Edwards H, Kwon KN, Morales C, Buckenmaier C' 3rd. Psychometric Testing of the Defense and Veterans Pain Rating Scale (DVPRS): A New Pain Scale for Military Population. Pain medicine (Malden, Mass.). 2016 Aug:17(8):1505-19. doi: 10.1093/pm/pnw105. Epub 2016 Jun 6     [PubMed PMID: 27272528]


[60]

Krebs EE, Lorenz KA, Bair MJ, Damush TM, Wu J, Sutherland JM, Asch SM, Kroenke K. Development and initial validation of the PEG, a three-item scale assessing pain intensity and interference. Journal of general internal medicine. 2009 Jun:24(6):733-8. doi: 10.1007/s11606-009-0981-1. Epub 2009 May 6     [PubMed PMID: 19418100]

Level 2 (mid-level) evidence

[61]

Kroenke K, Spitzer RL, Williams JB, Löwe B. An ultra-brief screening scale for anxiety and depression: the PHQ-4. Psychosomatics. 2009 Nov-Dec:50(6):613-21. doi: 10.1176/appi.psy.50.6.613. Epub     [PubMed PMID: 19996233]


[62]

Prins A, Bovin MJ, Smolenski DJ, Marx BP, Kimerling R, Jenkins-Guarnieri MA, Kaloupek DG, Schnurr PP, Kaiser AP, Leyva YE, Tiet QQ. The Primary Care PTSD Screen for DSM-5 (PC-PTSD-5): Development and Evaluation Within a Veteran Primary Care Sample. Journal of general internal medicine. 2016 Oct:31(10):1206-11. doi: 10.1007/s11606-016-3703-5. Epub 2016 May 11     [PubMed PMID: 27170304]


[63]

Friedrichsdorf SJ, Giordano J, Desai Dakoji K, Warmuth A, Daughtry C, Schulz CA. Chronic Pain in Children and Adolescents: Diagnosis and Treatment of Primary Pain Disorders in Head, Abdomen, Muscles and Joints. Children (Basel, Switzerland). 2016 Dec 10:3(4):     [PubMed PMID: 27973405]


[64]

Tomlinson D, von Baeyer CL, Stinson JN, Sung L. A systematic review of faces scales for the self-report of pain intensity in children. Pediatrics. 2010 Nov:126(5):e1168-98. doi: 10.1542/peds.2010-1609. Epub 2010 Oct 4     [PubMed PMID: 20921070]

Level 1 (high-level) evidence

[65]

Beyer JE, Denyes MJ, Villarruel AM. The creation, validation, and continuing development of the Oucher: a measure of pain intensity in children. Journal of pediatric nursing. 1992 Oct:7(5):335-46     [PubMed PMID: 1479552]

Level 1 (high-level) evidence

[66]

Breau LM, Camfield C, McGrath PJ, Rosmus C, Finley GA. Measuring pain accurately in children with cognitive impairments: refinement of a caregiver scale. The Journal of pediatrics. 2001 May:138(5):721-7     [PubMed PMID: 11343050]


[67]

Breau LM, McGrath PJ, Camfield CS, Finley GA. Psychometric properties of the non-communicating children's pain checklist-revised. Pain. 2002 Sep:99(1-2):349-57     [PubMed PMID: 12237214]


[68]

Malviya S, Voepel-Lewis T, Burke C, Merkel S, Tait AR. The revised FLACC observational pain tool: improved reliability and validity for pain assessment in children with cognitive impairment. Paediatric anaesthesia. 2006 Mar:16(3):258-65     [PubMed PMID: 16490089]


[69]

Solodiuk J, Curley MA. Pain assessment in nonverbal children with severe cognitive impairments: the Individualized Numeric Rating Scale (INRS). Journal of pediatric nursing. 2003 Aug:18(4):295-9     [PubMed PMID: 12923744]

Level 3 (low-level) evidence

[70]

Hunt A, Burne R. Medical and nursing problems of children with neurodegenerative disease. Palliative medicine. 1995 Jan:9(1):19-26     [PubMed PMID: 7719515]

Level 2 (mid-level) evidence

[71]

Voepel-Lewis T, Merkel S, Tait AR, Trzcinka A, Malviya S. The reliability and validity of the Face, Legs, Activity, Cry, Consolability observational tool as a measure of pain in children with cognitive impairment. Anesthesia and analgesia. 2002 Nov:95(5):1224-9, table of contents     [PubMed PMID: 12401598]

Level 1 (high-level) evidence

[72]

Keller S, Bann CM, Dodd SL, Schein J, Mendoza TR, Cleeland CS. Validity of the brief pain inventory for use in documenting the outcomes of patients with noncancer pain. The Clinical journal of pain. 2004 Sep-Oct:20(5):309-18     [PubMed PMID: 15322437]


[73]

Cleeland CS, Ryan KM. Pain assessment: global use of the Brief Pain Inventory. Annals of the Academy of Medicine, Singapore. 1994 Mar:23(2):129-38     [PubMed PMID: 8080219]


[74]

Dworkin RH, Turk DC, Revicki DA, Harding G, Coyne KS, Peirce-Sandner S, Bhagwat D, Everton D, Burke LB, Cowan P, Farrar JT, Hertz S, Max MB, Rappaport BA, Melzack R. Development and initial validation of an expanded and revised version of the Short-form McGill Pain Questionnaire (SF-MPQ-2). Pain. 2009 Jul:144(1-2):35-42. doi: 10.1016/j.pain.2009.02.007. Epub 2009 Apr 7     [PubMed PMID: 19356853]

Level 1 (high-level) evidence

[75]

Price DD. Psychological and neural mechanisms of the affective dimension of pain. Science (New York, N.Y.). 2000 Jun 9:288(5472):1769-72     [PubMed PMID: 10846154]

Level 3 (low-level) evidence

[76]

Gerhardt A, Hartmann M, Schuller-Roma B, Blumenstiel K, Bieber C, Eich W, Steffen S. The prevalence and type of Axis-I and Axis-II mental disorders in subjects with non-specific chronic back pain: results from a population-based study. Pain medicine (Malden, Mass.). 2011 Aug:12(8):1231-40. doi: 10.1111/j.1526-4637.2011.01190.x. Epub 2011 Aug 2     [PubMed PMID: 21810166]


[77]

Polatin PB, Kinney RK, Gatchel RJ, Lillo E, Mayer TG. Psychiatric illness and chronic low-back pain. The mind and the spine--which goes first? Spine. 1993 Jan:18(1):66-71     [PubMed PMID: 8434327]


[78]

Clark L, Jones K, Pennington K. Pain assessment practices with nursing home residents. Western journal of nursing research. 2004 Nov:26(7):733-50     [PubMed PMID: 15466611]


[79]

Closs SJ, Briggs M. Patients' verbal descriptions of pain and discomfort following orthopaedic surgery. International journal of nursing studies. 2002 Jul:39(5):563-72     [PubMed PMID: 11996877]


[80]

Long CJ. The relationship between surgical outcome and MMPI profiles in chronic pain patients. Journal of clinical psychology. 1981 Oct:37(4):744-9     [PubMed PMID: 6458625]

Level 2 (mid-level) evidence

[81]

BECK AT, WARD CH, MENDELSON M, MOCK J, ERBAUGH J. An inventory for measuring depression. Archives of general psychiatry. 1961 Jun:4():561-71     [PubMed PMID: 13688369]


[82]

Anekar AA, Hendrix JM, Cascella M. WHO Analgesic Ladder. StatPearls. 2024 Jan:():     [PubMed PMID: 32119322]


[83]

Azadfard M, Huecker MR, Leaming JM. Opioid Addiction. StatPearls. 2024 Jan:():     [PubMed PMID: 28846246]


[84]

Brott NR, Peterson E, Cascella M. Opioid, Risk Tool. StatPearls. 2024 Jan:():     [PubMed PMID: 31985940]


[85]

Webster LR. Risk Factors for Opioid-Use Disorder and Overdose. Anesthesia and analgesia. 2017 Nov:125(5):1741-1748. doi: 10.1213/ANE.0000000000002496. Epub     [PubMed PMID: 29049118]


[86]

Lee CS, Kim D, Park SY, Lee SC, Kim YC, Moon JY. Usefulness of the Korean Version of the CAGE-Adapted to Include Drugs Combined With Clinical Predictors to Screen for Opioid-Related Aberrant Behavior. Anesthesia and analgesia. 2019 Sep:129(3):864-873. doi: 10.1213/ANE.0000000000003580. Epub     [PubMed PMID: 31425231]


[87]

Atluri S, Akbik H, Sudarshan G. Prevention of opioid abuse in chronic non-cancer pain: an algorithmic, evidence based approach. Pain physician. 2012 Jul:15(3 Suppl):ES177-89     [PubMed PMID: 22786456]


[88]

Dowell D, Haegerich TM, Chou R. CDC Guideline for Prescribing Opioids for Chronic Pain--United States, 2016. JAMA. 2016 Apr 19:315(15):1624-45. doi: 10.1001/jama.2016.1464. Epub     [PubMed PMID: 26977696]


[89]

Williams ACC, Fisher E, Hearn L, Eccleston C. Psychological therapies for the management of chronic pain (excluding headache) in adults. The Cochrane database of systematic reviews. 2020 Aug 12:8(8):CD007407. doi: 10.1002/14651858.CD007407.pub4. Epub 2020 Aug 12     [PubMed PMID: 32794606]

Level 1 (high-level) evidence

[90]

Gibson W, Wand BM, Meads C, Catley MJ, O'Connell NE. Transcutaneous electrical nerve stimulation (TENS) for chronic pain - an overview of Cochrane Reviews. The Cochrane database of systematic reviews. 2019 Apr 3:4(4):CD011890. doi: 10.1002/14651858.CD011890.pub3. Epub 2019 Apr 3     [PubMed PMID: 30941745]

Level 3 (low-level) evidence

[91]

Rock AK, Truong H, Park YL, Pilitsis JG. Spinal Cord Stimulation. Neurosurgery clinics of North America. 2019 Apr:30(2):169-194. doi: 10.1016/j.nec.2018.12.003. Epub 2019 Feb 18     [PubMed PMID: 30898269]


[92]

Arendt-Nielsen L, Fernández-de-Las-Peñas C, Graven-Nielsen T. Basic aspects of musculoskeletal pain: from acute to chronic pain. The Journal of manual & manipulative therapy. 2011 Nov:19(4):186-93. doi: 10.1179/106698111X13129729551903. Epub     [PubMed PMID: 23115471]


[93]

Chou R, Qaseem A, Snow V, Casey D, Cross JT Jr, Shekelle P, Owens DK, Clinical Efficacy Assessment Subcommittee of the American College of Physicians, American College of Physicians, American Pain Society Low Back Pain Guidelines Panel. Diagnosis and treatment of low back pain: a joint clinical practice guideline from the American College of Physicians and the American Pain Society. Annals of internal medicine. 2007 Oct 2:147(7):478-91     [PubMed PMID: 17909209]

Level 3 (low-level) evidence

[94]

Zhang W, Moskowitz RW, Nuki G, Abramson S, Altman RD, Arden N, Bierma-Zeinstra S, Brandt KD, Croft P, Doherty M, Dougados M, Hochberg M, Hunter DJ, Kwoh K, Lohmander LS, Tugwell P. OARSI recommendations for the management of hip and knee osteoarthritis, Part II: OARSI evidence-based, expert consensus guidelines. Osteoarthritis and cartilage. 2008 Feb:16(2):137-62. doi: 10.1016/j.joca.2007.12.013. Epub     [PubMed PMID: 18279766]

Level 1 (high-level) evidence

[95]

Risser A, Donovan D, Heintzman J, Page T. NSAID prescribing precautions. American family physician. 2009 Dec 15:80(12):1371-8     [PubMed PMID: 20000300]


[96]

Hunter TS, Robison C, Gerbino PP. Emerging evidence in NSAID pharmacology: important considerations for product selection. The American journal of managed care. 2015 Apr:21(7 Suppl):S139-47     [PubMed PMID: 26168321]


[97]

Freynhagen R, Bennett MI. Diagnosis and management of neuropathic pain. BMJ (Clinical research ed.). 2009 Aug 12:339():b3002. doi: 10.1136/bmj.b3002. Epub 2009 Aug 12     [PubMed PMID: 19675082]


[98]

Derry S, Bell RF, Straube S, Wiffen PJ, Aldington D, Moore RA. Pregabalin for neuropathic pain in adults. The Cochrane database of systematic reviews. 2019 Jan 23:1(1):CD007076. doi: 10.1002/14651858.CD007076.pub3. Epub 2019 Jan 23     [PubMed PMID: 30673120]

Level 1 (high-level) evidence

[99]

Tremont-Lukats IW, Megeff C, Backonja MM. Anticonvulsants for neuropathic pain syndromes: mechanisms of action and place in therapy. Drugs. 2000 Nov:60(5):1029-52     [PubMed PMID: 11129121]


[100]

Skljarevski V, Zhang S, Iyengar S, D'Souza D, Alaka K, Chappell A, Wernicke J. Efficacy of Duloxetine in Patients with Chronic Pain Conditions. Current drug therapy. 2011 Nov:6(4):296-303     [PubMed PMID: 22876216]


[101]

Aiyer R, Barkin RL, Bhatia A. Treatment of Neuropathic Pain with Venlafaxine: A Systematic Review. Pain medicine (Malden, Mass.). 2017 Oct 1:18(10):1999-2012. doi: 10.1093/pm/pnw261. Epub     [PubMed PMID: 27837032]

Level 1 (high-level) evidence

[102]

Dworkin RH, O'Connor AB, Backonja M, Farrar JT, Finnerup NB, Jensen TS, Kalso EA, Loeser JD, Miaskowski C, Nurmikko TJ, Portenoy RK, Rice ASC, Stacey BR, Treede RD, Turk DC, Wallace MS. Pharmacologic management of neuropathic pain: evidence-based recommendations. Pain. 2007 Dec 5:132(3):237-251. doi: 10.1016/j.pain.2007.08.033. Epub 2007 Oct 24     [PubMed PMID: 17920770]

Level 3 (low-level) evidence

[103]

Cline AE, Turrentine JE. Compounded Topical Analgesics for Chronic Pain. Dermatitis : contact, atopic, occupational, drug. 2016 Sep-Oct:27(5):263-71. doi: 10.1097/DER.0000000000000216. Epub     [PubMed PMID: 27649348]


[104]

Patil S, Willett O, Thompkins T, Hermann R, Ramanathan S, Cornett EM, Fox CJ, Kaye AD. Botulinum Toxin: Pharmacology and Therapeutic Roles in Pain States. Current pain and headache reports. 2016 Mar:20(3):15. doi: 10.1007/s11916-016-0545-0. Epub     [PubMed PMID: 26879873]


[105]

Von Korff M, Kolodny A, Deyo RA, Chou R. Long-term opioid therapy reconsidered. Annals of internal medicine. 2011 Sep 6:155(5):325-8. doi: 10.7326/0003-4819-155-5-201109060-00011. Epub     [PubMed PMID: 21893626]


[106]

Els C, Jackson TD, Kunyk D, Lappi VG, Sonnenberg B, Hagtvedt R, Sharma S, Kolahdooz F, Straube S. Adverse events associated with medium- and long-term use of opioids for chronic non-cancer pain: an overview of Cochrane Reviews. The Cochrane database of systematic reviews. 2017 Oct 30:10(10):CD012509. doi: 10.1002/14651858.CD012509.pub2. Epub 2017 Oct 30     [PubMed PMID: 29084357]

Level 3 (low-level) evidence

[107]

Chou R, Fanciullo GJ, Fine PG, Adler JA, Ballantyne JC, Davies P, Donovan MI, Fishbain DA, Foley KM, Fudin J, Gilson AM, Kelter A, Mauskop A, O'Connor PG, Passik SD, Pasternak GW, Portenoy RK, Rich BA, Roberts RG, Todd KH, Miaskowski C, American Pain Society-American Academy of Pain Medicine Opioids Guidelines Panel. Clinical guidelines for the use of chronic opioid therapy in chronic noncancer pain. The journal of pain. 2009 Feb:10(2):113-30. doi: 10.1016/j.jpain.2008.10.008. Epub     [PubMed PMID: 19187889]


[108]

Frieden TR, Houry D. Reducing the Risks of Relief--The CDC Opioid-Prescribing Guideline. The New England journal of medicine. 2016 Apr 21:374(16):1501-4. doi: 10.1056/NEJMp1515917. Epub 2016 Mar 15     [PubMed PMID: 26977701]


[109]

Manchikanti L, Kaye AM, Knezevic NN, McAnally H, Slavin K, Trescot AM, Blank S, Pampati V, Abdi S, Grider JS, Kaye AD, Manchikanti KN, Cordner H, Gharibo CG, Harned ME, Albers SL, Atluri S, Aydin SM, Bakshi S, Barkin RL, Benyamin RM, Boswell MV, Buenaventura RM, Calodney AK, Cedeno DL, Datta S, Deer TR, Fellows B, Galan V, Grami V, Hansen H, Helm Ii S, Justiz R, Koyyalagunta D, Malla Y, Navani A, Nouri KH, Pasupuleti R, Sehgal N, Silverman SM, Simopoulos TT, Singh V, Solanki DR, Staats PS, Vallejo R, Wargo BW, Watanabe A, Hirsch JA. Responsible, Safe, and Effective Prescription of Opioids for Chronic Non-Cancer Pain: American Society of Interventional Pain Physicians (ASIPP) Guidelines. Pain physician. 2017 Feb:20(2S):S3-S92     [PubMed PMID: 28226332]


[110]

Sachse C, Brockmöller J, Bauer S, Roots I. Cytochrome P450 2D6 variants in a Caucasian population: allele frequencies and phenotypic consequences. American journal of human genetics. 1997 Feb:60(2):284-95     [PubMed PMID: 9012401]


[111]

Koopmans AB, Braakman MH, Vinkers DJ, Hoek HW, van Harten PN. Meta-analysis of probability estimates of worldwide variation of CYP2D6 and CYP2C19. Translational psychiatry. 2021 Feb 24:11(1):141. doi: 10.1038/s41398-020-01129-1. Epub 2021 Feb 24     [PubMed PMID: 33627619]

Level 1 (high-level) evidence

[112]

Tobias JD, Green TP, Coté CJ, SECTION ON ANESTHESIOLOGY AND PAIN MEDICINE, COMMITTEE ON DRUGS. Codeine: Time to Say "No". Pediatrics. 2016 Oct:138(4):. pii: e20162396. Epub 2016 Sep 19     [PubMed PMID: 27647717]


[113]

Schmidt-Hansen M, Bromham N, Taubert M, Arnold S, Hilgart JS. Buprenorphine for treating cancer pain. The Cochrane database of systematic reviews. 2015 Mar 31:2015(3):CD009596. doi: 10.1002/14651858.CD009596.pub4. Epub 2015 Mar 31     [PubMed PMID: 25826743]

Level 1 (high-level) evidence

[114]

Mukau L, Wormley K, Tomaszewski C, Ahmad B, Vohra R, Herring AA. Buprenorphine for High-dose Tramadol Dependence: A Case Report of Successful Outpatient Treatment. Clinical practice and cases in emergency medicine. 2024 Feb:8(1):81-82. doi: 10.5811/cpcem.4914. Epub     [PubMed PMID: 38546322]

Level 3 (low-level) evidence

[115]

Likar R, Krainer B, Sittl R. Challenging the equipotency calculation for transdermal buprenorphine: four case studies. International journal of clinical practice. 2008 Jan:62(1):152-6. doi: 10.1111/j.1742-1241.2007.01531.x. Epub     [PubMed PMID: 18173815]

Level 3 (low-level) evidence

[116]

Mercadante S, Casuccio A, Tirelli W, Giarratano A. Equipotent doses to switch from high doses of opioids to transdermal buprenorphine. Supportive care in cancer : official journal of the Multinational Association of Supportive Care in Cancer. 2009 Jun:17(6):715-8. doi: 10.1007/s00520-008-0546-6. Epub 2008 Dec 23     [PubMed PMID: 19104845]


[117]

Berkowitz BA. The relationship of pharmacokinetics to pharmacological activity: morphine, methadone and naloxone. Clinical pharmacokinetics. 1976:1(3):219-30     [PubMed PMID: 13957]


[118]

Nilsson MI, Meresaar U, Anggård E. Clinical pharmacokinetics of methadone. Acta anaesthesiologica Scandinavica. Supplementum. 1982:74():66-9     [PubMed PMID: 6953740]


[119]

Alinejad S, Ghaemi K, Abdollahi M, Mehrpour O. Nephrotoxicity of methadone: a systematic review. SpringerPlus. 2016:5(1):2087. doi: 10.1186/s40064-016-3757-1. Epub 2016 Dec 9     [PubMed PMID: 28018795]

Level 1 (high-level) evidence

[120]

Walker PW, Palla S, Pei BL, Kaur G, Zhang K, Hanohano J, Munsell M, Bruera E. Switching from methadone to a different opioid: what is the equianalgesic dose ratio? Journal of palliative medicine. 2008 Oct:11(8):1103-8. doi: 10.1089/jpm.2007.0285. Epub     [PubMed PMID: 18980450]

Level 2 (mid-level) evidence

[121]

Mercadante S, Caraceni A. Conversion ratios for opioid switching in the treatment of cancer pain: a systematic review. Palliative medicine. 2011 Jul:25(5):504-15. doi: 10.1177/0269216311406577. Epub     [PubMed PMID: 21708857]

Level 1 (high-level) evidence

[122]

Weschules DJ, Bain KT. A systematic review of opioid conversion ratios used with methadone for the treatment of pain. Pain medicine (Malden, Mass.). 2008 Jul-Aug:9(5):595-612. doi: 10.1111/j.1526-4637.2008.00461.x. Epub 2008 Jun 28     [PubMed PMID: 18565004]

Level 1 (high-level) evidence

[123]

Edwards JN, Whitney MA, Smith BB, Fah MK, Buckner Petty SA, Durra O, Sell-Dottin KA, Portner E, Wittwer ED, Milam AJ. The role of methadone in cardiac surgery for management of postoperative pain. BJA open. 2024 Jun:10():100270. doi: 10.1016/j.bjao.2024.100270. Epub 2024 Mar 22     [PubMed PMID: 38560623]


[124]

Yasaei R, Rosani A, Saadabadi A. Meperidine. StatPearls. 2024 Jan:():     [PubMed PMID: 29261967]


[125]

Cheatle MD, Savage SR. Informed consent in opioid therapy: a potential obligation and opportunity. Journal of pain and symptom management. 2012 Jul:44(1):105-16. doi: 10.1016/j.jpainsymman.2011.06.015. Epub 2012 Mar 21     [PubMed PMID: 22445273]


[126]

Helm S, Trescot AM, Colson J, Sehgal N, Silverman S. Opioid antagonists, partial agonists, and agonists/antagonists: the role of office-based detoxification. Pain physician. 2008 Mar-Apr:11(2):225-35     [PubMed PMID: 18354714]

Level 1 (high-level) evidence

[127]

Dayer P, Desmeules J, Collart L. [Pharmacology of tramadol]. Drugs. 1997:53 Suppl 2():18-24     [PubMed PMID: 9190321]

Level 3 (low-level) evidence

[128]

Rzasa Lynn R, Galinkin JL. Naloxone dosage for opioid reversal: current evidence and clinical implications. Therapeutic advances in drug safety. 2018 Jan:9(1):63-88. doi: 10.1177/2042098617744161. Epub 2017 Dec 13     [PubMed PMID: 29318006]

Level 3 (low-level) evidence

[129]

Theriot J, Sabir S, Azadfard M. Opioid Antagonists. StatPearls. 2024 Jan:():     [PubMed PMID: 30725764]


[130]

Smith HS. Opioid metabolism. Mayo Clinic proceedings. 2009 Jul:84(7):613-24. doi: 10.1016/S0025-6196(11)60750-7. Epub     [PubMed PMID: 19567715]


[131]

Bertilsson L, Dahl ML, Dalén P, Al-Shurbaji A. Molecular genetics of CYP2D6: clinical relevance with focus on psychotropic drugs. British journal of clinical pharmacology. 2002 Feb:53(2):111-22     [PubMed PMID: 11851634]


[132]

Jin J. Risks of Codeine and Tramadol in Children. JAMA. 2017 Oct 17:318(15):1514. doi: 10.1001/jama.2017.13534. Epub     [PubMed PMID: 29049655]


[133]

Orliaguet G, Hamza J, Couloigner V, Denoyelle F, Loriot MA, Broly F, Garabedian EN. A case of respiratory depression in a child with ultrarapid CYP2D6 metabolism after tramadol. Pediatrics. 2015 Mar:135(3):e753-5. doi: 10.1542/peds.2014-2673. Epub 2015 Feb 2     [PubMed PMID: 25647677]

Level 3 (low-level) evidence

[134]

Shaheen PE, Walsh D, Lasheen W, Davis MP, Lagman RL. Opioid equianalgesic tables: are they all equally dangerous? Journal of pain and symptom management. 2009 Sep:38(3):409-17. doi: 10.1016/j.jpainsymman.2009.06.004. Epub     [PubMed PMID: 19735901]


[135]

Syrmis W, Good P, Wootton J, Spurling G. Opioid conversion ratios used in palliative care: is there an Australian consensus? Internal medicine journal. 2014 May:44(5):483-9. doi: 10.1111/imj.12401. Epub     [PubMed PMID: 24589128]

Level 2 (mid-level) evidence

[136]

Reddy A, Vidal M, Stephen S, Baumgartner K, Dost S, Nguyen A, Heung Y, Kwan S, Wong A, Pangemanan I, Azhar A, Tayjasanant S, Rodriguez E, Waletich J, Lim KH, Wu J, Liu D, Williams J, Yennurajalingam S, Bruera E. The Conversion Ratio From Intravenous Hydromorphone to Oral Opioids in Cancer Patients. Journal of pain and symptom management. 2017 Sep:54(3):280-288. doi: 10.1016/j.jpainsymman.2017.07.001. Epub 2017 Jul 13     [PubMed PMID: 28711751]


[137]

Knotkova H, Fine PG, Portenoy RK. Opioid rotation: the science and the limitations of the equianalgesic dose table. Journal of pain and symptom management. 2009 Sep:38(3):426-39. doi: 10.1016/j.jpainsymman.2009.06.001. Epub     [PubMed PMID: 19735903]


[138]

Anderson R, Saiers JH, Abram S, Schlicht C. Accuracy in equianalgesic dosing. conversion dilemmas. Journal of pain and symptom management. 2001 May:21(5):397-406     [PubMed PMID: 11369161]


[139]

Vissers KC, Besse K, Hans G, Devulder J, Morlion B. Opioid rotation in the management of chronic pain: where is the evidence? Pain practice : the official journal of World Institute of Pain. 2010 Mar-Apr:10(2):85-93. doi: 10.1111/j.1533-2500.2009.00335.x. Epub 2010 Jan 8     [PubMed PMID: 20070552]


[140]

Matteliano D, St Marie BJ, Oliver J, Coggins C. Adherence monitoring with chronic opioid therapy for persistent pain: a biopsychosocial-spiritual approach to mitigate risk. Pain management nursing : official journal of the American Society of Pain Management Nurses. 2014 Mar:15(1):391-405. doi: 10.1016/j.pmn.2012.08.008. Epub 2012 Oct 23     [PubMed PMID: 24602442]


[141]

Tunks ER, Crook J, Weir R. Epidemiology of chronic pain with psychological comorbidity: prevalence, risk, course, and prognosis. Canadian journal of psychiatry. Revue canadienne de psychiatrie. 2008 Apr:53(4):224-34     [PubMed PMID: 18478825]


[142]

Watkins PB, Kaplowitz N, Slattery JT, Colonese CR, Colucci SV, Stewart PW, Harris SC. Aminotransferase elevations in healthy adults receiving 4 grams of acetaminophen daily: a randomized controlled trial. JAMA. 2006 Jul 5:296(1):87-93     [PubMed PMID: 16820551]

Level 1 (high-level) evidence

[143]

Holubek WJ, Kalman S, Hoffman RS. Acetaminophen-induced acute liver failure: results of a United States multicenter, prospective study. Hepatology (Baltimore, Md.). 2006 Apr:43(4):880; author reply 882     [PubMed PMID: 16557558]

Level 3 (low-level) evidence

[144]

Jalan R, Williams R, Bernuau J. Paracetamol: are therapeutic doses entirely safe? Lancet (London, England). 2006 Dec 23:368(9554):2195-6     [PubMed PMID: 17189017]


[145]

Cavalcante AN, Sprung J, Schroeder DR, Weingarten TN. Multimodal Analgesic Therapy With Gabapentin and Its Association With Postoperative Respiratory Depression. Anesthesia and analgesia. 2017 Jul:125(1):141-146. doi: 10.1213/ANE.0000000000001719. Epub     [PubMed PMID: 27984223]


[146]

Lee M, Silverman SM, Hansen H, Patel VB, Manchikanti L. A comprehensive review of opioid-induced hyperalgesia. Pain physician. 2011 Mar-Apr:14(2):145-61     [PubMed PMID: 21412369]


[147]

Sansone RA, Sansone LA. Doctor shopping: a phenomenon of many themes. Innovations in clinical neuroscience. 2012 Nov:9(11-12):42-6     [PubMed PMID: 23346518]


[148]

Lopez MJ, Preuss CV, Tadi P. Drug Enforcement Administration Drug Scheduling. StatPearls. 2024 Jan:():     [PubMed PMID: 32491358]


[149]

Gabay M. The federal controlled substances act: schedules and pharmacy registration. Hospital pharmacy. 2013 Jun:48(6):473-4. doi: 10.1310/hpj4806-473. Epub     [PubMed PMID: 24421507]


[150]

Larrat EP, Marcoux RM, Vogenberg FR. Implications of recent controlled substance policy initiatives. P & T : a peer-reviewed journal for formulary management. 2014 Feb:39(2):126-8     [PubMed PMID: 24669180]


[151]

Gabay M. Federal controlled substances act: controlled substances prescriptions. Hospital pharmacy. 2013 Sep:48(8):644-5. doi: 10.1310/hpj4808-644. Epub     [PubMed PMID: 24421533]


[152]

Preuss CV, Kalava A, King KC. Prescription of Controlled Substances: Benefits and Risks. StatPearls. 2024 Jan:():     [PubMed PMID: 30726003]


[153]

Weyandt LL, Oster DR, Marraccini ME, Gudmundsdottir BG, Munro BA, Rathkey ES, McCallum A. Prescription stimulant medication misuse: Where are we and where do we go from here? Experimental and clinical psychopharmacology. 2016 Oct:24(5):400-414. doi: 10.1037/pha0000093. Epub     [PubMed PMID: 27690507]


[154]

Clinton HA, Hunter AA, Logan SB, Lapidus GD. Evaluating opioid overdose using the National Violent Death Reporting System, 2016. Drug and alcohol dependence. 2019 Jan 1:194():371-376. doi: 10.1016/j.drugalcdep.2018.11.002. Epub 2018 Nov 15     [PubMed PMID: 30481691]


[155]

Rose AJ, McBain R, Schuler MS, LaRochelle MR, Ganz DA, Kilambi V, Stein BD, Bernson D, Chui KKH, Land T, Walley AY, Stopka TJ. Effect of Age on Opioid Prescribing, Overdose, and Mortality in Massachusetts, 2011 to 2015. Journal of the American Geriatrics Society. 2019 Jan:67(1):128-132. doi: 10.1111/jgs.15659. Epub 2018 Nov 24     [PubMed PMID: 30471102]


[156]

Jones CM, Lurie PG, Throckmorton DC. Effect of US Drug Enforcement Administration's Rescheduling of Hydrocodone Combination Analgesic Products on Opioid Analgesic Prescribing. JAMA internal medicine. 2016 Mar:176(3):399-402. doi: 10.1001/jamainternmed.2015.7799. Epub     [PubMed PMID: 26809459]


[157]

Drug Enforcement Administration, Department of Justice. Schedules of controlled substances: rescheduling of hydrocodone combination products from schedule III to schedule II. Final rule. Federal register. 2014 Aug 22:79(163):49661-82     [PubMed PMID: 25167591]


[158]

Chen JH, Humphreys K, Shah NH, Lembke A. Distribution of Opioids by Different Types of Medicare Prescribers. JAMA internal medicine. 2016 Feb:176(2):259-61. doi: 10.1001/jamainternmed.2015.6662. Epub     [PubMed PMID: 26658497]


[159]

Jones CM, Compton W, Vythilingam M, Giroir B. Naloxone Co-prescribing to Patients Receiving Prescription Opioids in the Medicare Part D Program, United States, 2016-2017. JAMA. 2019 Aug 6:322(5):462-464. doi: 10.1001/jama.2019.7988. Epub     [PubMed PMID: 31386124]